Nothing Special   »   [go: up one dir, main page]

PARTNERSHIP

Download as docx, pdf, or txt
Download as docx, pdf, or txt
You are on page 1of 84

Chapter 1

PARTNERSHIP

INTRODUCTION

There are no authoritative pronouncements concerning the accounting for partnership; thus all
principles described herein have evolved through accounting practice.

PARTNERSHIP FORMATION

The partnership is a separate accounting entity, and therefore, its assets and liabilities should
remain separate and distinct from the individual partner's personal assets and liabilities. Thus, all
assets contributed to the partnership are recorded in the books of the partnership at their fair
market values, and all liabilities assumed by the partnership are recorded at their present values.

The accounting problems peculiar to a partnership relate to the measurement of the individual
partners' ownership equities or interests in the partnership. A partner's interest in a firm should
be distinguished from the right to share in firm's profits. A partner's interest is summarized in an
individual capital account and consists of the original investment, subsequent additional
investments and withdrawals, and the partner's share of firm's profits and losses.

ALLOCATION OF PARTNERSHIP INCOME OR LOSS

Partners may agree to share profits and losses in any manner, irrespective of capital interests.
This agreement should be stipulated in the articles of partnership. In the absence of an
expressed agreement, profits and losses shall be divided based on the contributed capital of the
partners.

Generally, the division of partnership income should be based on an analysis of the correlation
between the capital and labor committed to the firm by individual partners and the income that
subsequently is generated. As a result, profits might be divided in one or more of the following
ways:
Profit and Loss Ratio

This method obviously provides a simplified way and equitable division of profits. Normally, the
ratio designed for the division of profits also is used for the division of losses, unless a specific
provision to the contrary exists.

Capital Investment of Partners

The division of the portion of the profit using the capital investment of partners may be
accomplished by imputing interest on the invested capital at some specified rate. This interest is
not viewed as a partnership expense, but rather as a means of allocating profits and losses
among partners. When this approach will be used, the partnership agreement should specify the
partners' capital investment whether the capital balances are before or after their respective
withdrawals; capital investment at the beginning or end of an accounting period or the weighted
average capital during the period.

Services Rendered by Partners

Normally, the profit and loss agreement recognizes variations in effort by calling for a portion of
income to be allocated to partners as salaries, which, like interest on capital investments, are
viewed as a means of allocating income rather than as an expense. Thus, this treatment of
partners' salaries differs from the treatment of employee/shareholder salaries in a corporation.

To reward partners' services to the partnership beyond that already recognized by salaries
and/or interest, bonuses may be provided to partner/s. The bonus may be expressed as a
percentage of partnership net income before or after bonus.

Allocation of Profit Deficiencies and Losses

When the partnership income was large enough to satisfy all of the provisions of the profit and
loss agreement, the allocation of profit may be simple; however if the income is not sufficient or
an operating loss exists, one of the following alternatives may be used:

1. Completely satisfy all provisions of the profit and loss agreement and use the profit and
loss ratios to absorb any deficiency or additional loss caused by such action.

2. Satisfy each of the provisions to whatever extent it is possible. In other words, satisfy
each
of the provisions based on the order of priority agreed by the partners.
PARTNERSHIP DISSOLUTION (CHANGES IN OWNERSHIP)

A partnership is said to be dissolved when the original association for purposes of carrying on
activities has ended. Although dissolution brings to an end the association of individuals for their
original purpose, it does not mean the termination of business or even an interruption of its
continuity. Upon the death or retirement of a partner, the business may continue as a new
partnership composed of the remaining partners. This is not to be confused with partnership
liquidation, which is the winding up of partnership affairs and termination of the business. In
short, under dissolution, the partnership business continues, but under different ownership.

When partnership dissolution occurs, a new accounting entity results, therefore, the partnership,
after allocation of income or loss to the existing partners' capital accounts, should adjust all
assets and liabilities to their fair market values and present values, respectively.

After all adjustments have been made, the accounting for dissolution depends on the type of
transaction that caused the dissolution, such as:

1. Transactions between the partnership and a partner (e.g. a new partner contributes
assets, or a retiring partner withdraws assets).
2. Transactions between partners (e.g. a new partner purchases an interest from one or
more existing partners, or a retiring partner sells his interest to one or more existing
partners).

Transactions Between a Partner and the Partnership

1. Admission of a New Partner


When a new partner is admitted to the partnership, the new partner can invest assets into the
partnership and receive a capital balance equal or greater/less than to the assets invested. If
the new partner's capital balance is not equal to the assets invested, then either the bonus or
goodwill method must be used to account for the difference.

Bonus Method

The bonus method adheres to the historical cost concept, thus, strictly observes that
net assets should be recorded at historical cost. Any increase in the value of net assets
should not be recognized until they are realized in an actual exchange transaction;
however, write-downs in the value of net assets may be recognized.
Under this method, the total contributed capital (including that of the new partner) is
equal to the new partnership capital. The bonus method implies that the old partners
either received a bonus from the new partner, or they paid bonus to the new partner.

Bonus to the Old Partner

The book value approach of the bonus method does not recognize increase in
net asset values or recognition of goodwill. Therefore, when an incoming
partner's contribution indicates the existence of which, bonus may be granted
to the old partners. This will eventually increase the capital accounts of the old
partners and reducing the new partner's capital account.

Bonus to the New Partner

When the incoming partner invests some intangible assets to the partnership,
bonus may be credited to him: This may be viewed as a cost incurred to
acquire the intangible assets contributed by the incoming partner, since all
costs to acquire assets eventually affect income and are allocated among the
partners.

Goodwill Method

This method emphasizes the legal significance of a change in the capital structure of a
partnership. From a legal point of view, the admission of a new partner results in the
dissolution of the previous partnership and the creation a new legal entity. Since a new
legal entity exists, assets contributed to
form this new entity should be recorded at their fair market value and eventually
recognize asset appreciation or goodwill.
Under this method, the total contributed capital (including that of the new partner) is
less than the new partnership capital. Goodwill may be traced to the new or old
partners, if not all partners.

Goodwill to the Old Partners

In the event that there are no differences between the fair value and book
value
of recorded assets, unrecorded goodwill may be recognized. To the mere fact
that the new partier is willing to pay more than the proportionate book value
of the new entity, this indicates that goodwill existed prior to the new partner's
admission. Thus, goodwill is recorded and allocated to the old partners.
Goodwill to the New Partner

In some cases, a partnership may be in urgent need of additional funds or the


partners may desire the services of a certain individual, a new partner may be
admitted with the provision that bonus or goodwill may be provided to him.

2. Partner's Death or Withdrawal

The death or withdrawal of a partner is treated in a manner similar with the admission of a
new partner. However, there is no new capital account to be recorded. Under this situation,
either bonus or goodwill method may be used. It is important to note that in regard to a
partner's withdrawal from the partnership, the capital account of the withdrawing partner
must be adjusted to the amount that the withdrawing partner is expected to receive.

Transactions Between Partners

The sale of a partnership interest is a transaction between the partners only, and the partners
involved may simply transfer a portion of the existing partners' capital to a new capital account
for the buying partner, or an implied goodwill will be recorded. It should be noted that no journal
entry is made to record the receipt of cash because the cash goes directly to the selling
partner/s, and after the purchase, the capital balance of the newly admitted partner is equal to
the amount of the purchase price.

PARTNERSHIP LIQUIDATION

Liquidation is the winding up of the partnership business, which is the selling of its all noncash
assets, payment of its liabilities, and final settlement to the remaining partners.

The four basic steps to partnership liquidation are:

1. Net income or loss up to the date of liquidation should be allocated to the partners'
capital
accounts based on their P&L ratio.
2. The gain or loss realized from the sale of noncash assets should be allocated to the
partners' capital accounts based on their P&L ratio.
3. Creditors' claims, including liquidation expenses or anticipated future claims should be
paid.
4. Remaining cash is distributed to partners in accordance with the balance in their capital
accounts, and not the P&L ratio.
It should be noted that when loans exist between the partnership and a partner, the capital
account and the loan/s are combined (right of offset). When a deficit exists and the deficient
partner is insolvent, the amount of the deficit is allocated to the remaining solvent partners'
capital accounts on the basis of their P&L ratio. Note that if the partner with the deficit is
personally solvent, he has a liability to the remaining partners for the amount of the deficit.

A partnership may be liquidated by either lump-sum liquidation or liquidation by installment.

Lump-sum Liquidation

A lump-sum liquidation is one in which all of the assets are sold in bulk and all of the creditors'
claims are satisfied before a single liquidating distribution is made to the partners. Because the
assets are sold in bulk, there is a tendency to realize greater losses than if the assets were sold
over a period of time.

Installment Liquidation

Under this form of liquidation, the assets are sold over a period of time and the cash is
distributed to the partners as it becomes available. There are two keys to prepare a statement of
partnership liquidation under the installment liquidation, namely:

1. Determination of the Available Cash Balance.

The beginning cash balance, which is the balance before liquidation, is adjusted for the
cash receipts from receivables, sale of noncash assets, payments to creditors, and
liquidation expenses incurred. A situation may occur where a certain amount of cash is
to be reserved for payment of future liabilities that may arise. If so, this cash should be
treated as a non-cash assets which makes it unavailable for current distribution to the
partners.

2. Determination of which Partner will Receive the Available Cash.


There are a number of ways to make this computation, all of which are equally correct
in the eyes of the examiners. This determination can be made at the beginning of the
liquidation process (Cash Priority Program) or at the time of each payment (Schedule of
Safe Payment).
Cash Priority Program
It is in installment distribution plan prepared before the actual liquidation
which permits the partners to determine how cash should be distributed safely
if and when it becomes available. Under this program, to determine the order
of distributions and amount of payments (per priority), the loss absorption
balance of each partner must be computed. The loss absorption balance is the
maximum loss each partner can absorb and which can eliminate him from any
cash distribution.

Schedule of Safe Payment

It is a schedule of installment payment of cash to partners prepared


periodically as if no more cash is forthcoming. Under this schedule cash is
distributed to a partner only if he has an excess credit balance in his
partnership interest after absorption of his share of the maximum possible loss
(total value of unsold noncash assets plus cash withheld for payment of future
obligations) that may occur.

INCORPORATION OF A PARTNERSHIP

The incorporation of a partnership results in the formation of a new accounting and legal entity,
which means the adjustments of the partnership records up to the date of incorporation. The
partnership closes its books and recognizes any income or loss up to the date of incorporation,
and the books of the partnership are adjusted to reflect the fair market value of the partnership
assets and the present value of the partnership liabilities; accordingly, adjustment is made to the
capital accounts in accordance with the partners profit & loss ratio. Finally, common stock is
distributed to the partners in accordance with the amounts in their capital accounts.
MULTIPLE CHOICE QUESTIONS

PROB. 1-1 (AICPA)

Which of the following is not a characteristic of most partnership?


a. Limited liability
b. Limited life
c. Mutual agency
d. Ease of formation

PROB. 1-2 (AICPA)

Which of the following is not a characteristic of the proprietary theory that influences
accounting for partnerships?
a. Partners' salaries are viewed as a distribution of income rather than a component of net
income.
b. A partnership is not viewed as separate entity, distinct, taxable entity.
c. A partnership is characterized by limited liability.
d. Changes in the ownership structure of a partnership result in the dissolution of the
partnership.

PROB. 1-3 (AICPA)

Which of the following statements is correct with respect to a limited partnership?


a. A limited partner may not be an unsecured creditor of the limited partnership.
b. A general partner may not also be limited partner at the same time.
c. A general partner may be a secured creditor of the limited partnership
d. A limited partnership can be formed with limited liability for all partners.

PROB. 1-4 (AICPA)

An advantage of the partnership as a form of business organization would be


a. Partners do not pay income taxes on their share in partnership income.
b. A partnership is bound by the act of the partners.
c. A partnership is created by mere agreements of the partners.
d. A partnership may be terminated by the death or withdrawal of a partner.
PROB. 1-5 (AICPA)

When property other than cash is invested in a partnership, at what amount should the
noncash property be credited to the contributing partner's capital account?
a. Fair value at the date of contribution.
b. Contributing partner's original cost.
c. Assessed valuation for property tax purposes.
d. Contributing partner's tax basis.

PROB. 1-6 (Adapted)

A and B formed a partnership, each contributing non-cash assets into the partnership.
Partner A contributed inventory with a current market value in excess of its carrying
amount. Partner B contributed fixed asset with a carrying amount in excess of its current
market value. At what amount should the partnership record each of the assets
contributed?
Inventory Fixed Asset
a. Carrying Amount Market Value
b Market Value Carrying Amount
.
c. Carrying Amount Carrying Amount
d Market Value Market Value
.
PROB. 1-7 (Adapted)

Partnership capital and drawings accounts are similar to the corporate


a. Paid in capital, retained earnings, and dividends accounts.
b. Retained earnings account.
c. Paid in capital and retained earnings accounts.
d. Preferred and common stock accounts.

PROB. 1-8 (Adapted)

On April 30, 2016, AI, Ben, and Ces formed a partnership by combining their separate
business proprietorships. Al contributed cash of P50,000. Ben contributed property with a
P36,000 carrying amount, a P40,000 original cost, and P80,000 fair value. The partnership
accepted responsibility for the P35,000 mortgage attached to the property. Ces contributed
equipment with a P30,000 carrying amount, a P75,000 original cost, and P55,000 fair value.
The

partnership agreement specifies that profits and losses are to be shared equally but is silent
regarding capital contributions. Which partner has the largest capital account balance at
April 30, 2016?
a. Al
b. Ben
c. Ces
d. All capital balances are equal

PROB. 1-9 (Adapted)

Al, Sharif, and Booba formed a partnership. Al will contribute cash of P50,000 and his store
equipment that originally cost P60,000 with a second-hand value of P25,000. Sharif will
contribute P80,000 in cash. Booba, whose family sells computers, will contribute P25,000
cash and a brand new computer that cost his family's computer dealership P50,000, but
with a regular selling price of P60,000. They agreed to share profits and losses equally. Upon
formation, what are the capital balances of the partners?
Al Sharif Booba
a. 75,000 80,000 85,000
b. 80,000 80,000 80,000
c. 88,333 88,333 88,333
d. 110,000 80,000 75,000

PROB. 1-10 (Adapted)


On January 1, 2016, Atta and Boy agreed to form a partnership contributing their respective
assets and equities subject to adjustments. On that date, the following were provided:
Atta Boy
Cash P 28,000 P 62,000
Accounts receivable 200,000 600,000
Inventories 120,000 200,000
Land 600,000
Building 500,000
Furniture & fixtures 50,000 35,000
Intangible assets 2,000 3,000
Accounts payable 180,000 250,000
Other liabilities 200,000 350,000
Capital 620,000 800,000

The following adjustments were agreed upon:


a. Accounts receivable of P20,000 and P40,000 are uncollectible in A's and B's respective
books.

b. Inventories of P6,000 and P7,000 are worthless in A's and B's respective books.

c. Intangible assets are to be written off in both books.

What will be the capital balances of the partners after adjustments?


Atta Boy
a. 592,000 750,000
b 600,000 700,000
.
c. 592,000 756,300
d 600,000 750,000
.

PROB. 1-11 (Adapted)

Mary admits Jane as a partner in the business. Balance sheet accounts of Mary just before
the admission of Jane show: Cash, P26,000, Accounts receivable, P120,000, Merchandise
inventory, P180,000, and Accounts payable, P62,000. It was agreed that for purposes of
establishing Mary's interest, the following adjustments be made: 1.) an allowance for
doubtful accounts of 3% of accounts receivable is to be established; 2.) merchandise
inventory is to be adjusted upward by P25,000; and 3.) prepaid expenses of P3,600 and
accrued liabilities of P4,000 are to be recognized.
If Jane is to invest sufficient cash to obtain 2/5 interest in the partnership, how much would
Jane contribute to the new partnership?
a. 176,000
b. 190,000
c. 95,000
d. 113,980
PROB. 1-12 (AICPA)

Roberts and Smith drafted a partnership agreement that lists the following assets
contributed at the partnership's formation:
Contributed by
Roberts Smith
Cash P20,000 P30,000
Inverntory 15,000
Building 40,000
Furniture & equipment 15,000

The building is subject to a mortgage of P10,000, which the partnership has assumed. The
partnership agreement also specifies that profits and losses are to be distributed evenly.
What amounts should be recorded as capital for Roberts and Smith at the formation of the
partnership?
Roberts Smith
a. 35,000 85,000
b. 35,000 75,000
c. 55,000 55,000
d. 60,000 60,000

PROB. 1- 13 (AICPA)

On May 1, 2016, Cobb and Mott formed a partnership and agreed to share profits and
losses in the ratio of 3:7, respectively. Cobb contributed a parcel of land that cost him
P10,000. Mott contributed P40,000 cash. The land was sold for P18,000 on May 1, 2016,
immediately after formation of the partnership. What amount should be recorded in Cobb's
capital account on formation of the partnership?
a. 18,000
b. 17,400
c. 15,000
d. 10,000

PROB. 1-14 (AICPA)


The Grey and Redd Partnership was formed on January 2, 2016, Under the partnership
agreement, each partner has an equal initial capital balance. Partnership net income or loss
is allocated 60% to Grey and 40% to Redd. To form the partnership, Grey originally
contributed assets costing P30,000 with a fair value of P60,000 on January 2, 2016, and
Redd contributed P20,000 cash. Drawings by the partners during 2016 totaled P3,000 by
Grey and P9,000 by Redd. The partnership net income in 2016 was P25,000.

a. Under the goodwill method, what is Redd's initial capital balance in the partnership?
a. 20,000
b. 25,000
c. 40,000
d. 60,000
b. Under the bonus method, what is the amount of bonus?
a. 20,000 bonus to Grey
b. 20,000 bonus to Redd
c. 40,000 bonus to Grey
d. 40,000 bonus to Redd

PROB. 1-15 (AICPA)

Abel and Carr formed a partnership and agreed to divide initial capital equally, even though
Abel contributed P100,000 and Carr contributed P84,000 in identifiable assets. Under the
bonus approach to adjust the capital accounts, Carr's unidentifiable asset should be debited
for
46,000
16,000
8,000
0

PROB. 1- 16 (Adapted)

On April 30, 2016, Alex, Benjie, and Cesar formed a partnership by combining their separate
business proprietorships. Alex contributed cash of P500,000. Benjie contributed property
with a P360,000 carrying amount, a P400,000 original cost, and P800,000 fair market value.

The partnership accepted responsibility for the P350,000 mortgage attached to the
property. Cesar contributed equipment with a P300,000 carrying amount, a P750,000
original cost, and P550,000 fair value. The partnership agreement specifies that profits and
losses are to be shared equally but is silent regarding capital contributions. What are the
capital balances of the partners at April 30, 2016?
Alex Benjie Cesar
a. 500,000 800,000 550,000
b. 500,000 450,000 550,000
c. 500,000 360,000 300,000
d. 500,000 400,000 750,000

PROB. 1- 17 (Adapted)

On January 2, 2016, Abel, Cain, and Josuah formed a partnership. Abel contributed cash of
P100,000 and a delivery equipment that originally costs him P120,000, but with a second
hand value of P50,000. Cain contributed P160,000 in cash. Josuah, whose family sells office
equipment, contributed P50,000 in cash and office equipment that cost his family's
dealership P100,000 but with a regular selling price of P120,000. In 2016, the partnership
reported net income of P120,000.

On December 31, 2016, what would be the capital balance of the partners?
Abel Cain Josuah
a. 257,500 200,000 192,500
b. 190,000 200,000 210,000
c. 260,000 200,000 190,000
d. 187,500 200,000 212,500

PROB. 1-18 (AICPA)

The Flat and Iron partnership agreement provides for Flat to receive a 20% bonus on profits
before bonus. Remaining profits and losses are divided between Flat and Iron in the ratio of
2:3, respectively. Which partner has a greater advantage when the partnership has a profit
or when it has a loss?

Profit Loss
a. Flat Iron
b. Flat Flat
c. Iron Flat
d. Iron Iron
PROB. 1-19 (Adapted)

Partners A and B share profits and losses equally after each has been credited in all
circumstances with annual salary allowances of P30,000 and P24,000, respectively. Based
on this agreement, in which of the following circumstances will Partner A benefit by P6,000
more than Partner B?
a. Only if the partnership has net income of P54,000 or more for the year.
b. Only if the partnership does not incur a loss for the year
c. In all earnings or loss situation.
d. Only if the partnership has earnings of at least P6,000 for the year.

PROB. 1-20 (AICPA)

Downs, Frey, and Vick formed the DFV general partnership to act as manufacturer's
representatives. The partners agreed Downs would receive 40% of any partnership profits
and Frey and Vick would each receive 30% of such profits. It was also agreed that the
partnership would not terminate for 5 years. After the fourth year, the partners agreed to
terminate the partnership. At that time, the partners capital accounts were as follows:
Downs, P20,000; Frey, P15,000; and Vick P10,000. There also were undistributed losses of
P30,000. Vick's share of the undistributed losses will be
a. 0
b. 1,000
c. 9,000
d. 10,000

PROB. 1- 21 (AICPA)
The partnership agreement of Reid and Simm provides that interest at 10% per year is to be
credited to each partner on the basis of weighted average capital balances. A summary of
Simm's capital account for the year-ended December 31, 2016, is as follows:

Balance, January 1 P 140,000


Additional investment, July 1 40,000
Withdrawal, August 1 (15,000)
Balance, December 31 165,000

PROB. 1-22 (AICPA)

Partner Ae first contributed P50,000 of capital into existing partnership on March 1, 2016.
On June 1, 2016, said partner contributed another P20,000. On September 1, 2016, he
withdrew P15,000 from the partnership. Withdrawal in excess of P10,000 are charged to
the partner's capital accounts. What is the annual weighted average capital balance of
Partner Ae?
a. 32,500
b. 51,667
c. 60,000
d. 48,333

PROB. 1-23 (Adapted)

If the partnership agreement does not specify how income is to be allocated, profits and
loss should be allocated
a. Equally.
b. In proportion to the weighted average of capital invested during the period.
c. Equitably so that partners are compensated for the time and effort expended on behalf
of the partnership.
d. In accordance with their capital contribution.

PROB. 1-24 (Adapted)

Which of the following is not a component of the formula used to distribute income?
a. Salary allocation to those partners working.
b. After all other allocation, the remainder divided according to the profit and loss sharing
ratio.
c. Interest on the average capital investments.
d. Interest on notes to partners.

PROB. 1-25 (Adapted)

Which of the following is not considered a legitimate expense of a partnership?


a. Interest paid to partners based on the amount of invested capital.
b. Depreciation on assets contributed to the partnership by partners.
c. Salaries for management hired to run the business.
d. Supplies used in the partners' offices.

PROB. 1-26 (AICPA)

The fact that salaries paid to partners are not a component of partnership income is
indicative of
a. A departure from generally accepted accounting principles.
b. Being characteristic of the entity theory.
c. Being characteristic of the proprietary theory.
d. Why partnerships are characterized by unlimited liability

PROB. 1-27 (AICPA)

The ABC Partnership reports net income of P60,000. If Partners A, B, and C have income
ratio of 50%, 30%, and 20%, respectively. What is the share of Partner C from the net
income of the partnership, if he was given a capital ratio of 25%?
a. 30,000
b. 12,000
c. 18,000
d. 15,000

PROB. 1- 28 (RPCPA)

In the calendar year 2016, the partnership of A and B realized a net profit of P240,000. The
capital accounts of the partners show the following postings:
A, Capital B, Capital
Debit Credit Debit Credit
Jan. 1 P120,000 P80,000
May 1 P20,000 P10,000
July 1 20,000
Aug. 1 10,000
Oct. 1 10,000 5,000

a. If the profits are to be divided based on average capital, the share of A and B,
respectively are:
a. 129,600 110,400
b. 144,000 96,000
c. 136,800 103,200
d. 136,543 103,457

b. If 20% interest based on the capital at the end of the year is allowed and given and the
balance of the P240,000 profit is divided equally, the total share of A and B, respectively
are:
a. 121,500 118,500
b. 124,000 116,000
c. 123,000 117,000
d. 122,625 117,375

PROB. 1-29 (AICPA)

During 2016, Young and Zinc maintained average capital balances in their partnership of
P160,000 and P100,000, respectively. The partners receive 10% interest on average capital
balances, and residual profit or loss is divided equally. Partnership profit before interest was
P4,000. By what amount should Zinc's capital account change for the year?
a. 1,000 decrease
b. 2,000 increase
c. 11,000 decrease
d. 12,000 increase

PROB. 1-30 (AICPA)

Red and White formed a partnership in 2016. The partnership agreement provides for
annual salary allowances of P55,000 for Red and P45,000 for White. The partners share
profits equally and losses in a 60/40 ratio. The partnership had earnings of P80,000 for 2016
before any allowance to partners. What amount of these earnings should be credited to
each partner's capital account?
Red White
a. 40,000 40,000
b. 43,000 37,000
c. 44,000 36,000
d. 45,000 35,000

PROB. 1-31 (AICPA)

Fox, Greg, and Howe are partners with average capital balances during 2016 of P120,000,
P60,000, and P40,000, respectively. Partners receive 10% interest on their average capital
balances. After deducting salaries of P30,000 to Fox and P20,000 to Howe, the residual
profit and loss is divided equally. In 2016, the partnership sustained a P33,000 loss before
interest and salaries to partners. By what amount should Fox's capital account change?
a. 7,000 increase
b. 11,000 decrease
c. 35,000 decrease
d. 42,000 increase

PROB. 1-32 (Adapted)

If a partnership has net income of P44,000 and Partner X is to be allocated bonus receive?
of 10% of income after the bonus. What is the amount of bonus Partner X will receive?
a. 3,000
b. 3,300
c. 4,000
d. 4,400

PROB. 1- 33 (AICPA)

The partnership agreement of Donn, Eddy, and Farr provides for annual distribution of
profit and loss in the following sequence:
 Donn, the managing partner, receives a bonus of 10% of profit.
 Each partner receives 6% interest on average capital investment.
 Residual profit or loss is divided equally.
Average capital investments for 2016 were:
Donn P80,000
Eddy 50,000
Farr 30,000

What portion of the P100,000 partnership profit for 2016 should be allocated to Farr?
a. 28,600
b. 29,800
c. 35,133
d. 41,600

PROB. 1-34 (Adapted)

The Articles of Partnership of Adam and Eve the following provisions were stipulated:
a. Annual salary of P60,000 each.
b. Bonus to Adam of 20% of the net income after partners' salaries, the bonus being
treated as an expense.
c. Balance to be divided equally.

The partnership reported a net income of P360,000 after partners' salaries but before
bonus. How much is the share of Eve in the profit?
60,000
90,000
150,000
210,000

PROB. 1-35 (Adapted)

Partners AA and BB have profit and loss agreement with the following provisions: salaries of
P30,000 and P45,000 for AA and BB, respectively; a bonus to AA of 10% of net income after

salaries and bonus; and interest of 10% on average capital balances of P20,000 and P35,000
for AA and BB, respectively. One-third of any remaining profits will be allocated to AA and
the balance to BB. If the partnership had net income of P102,500, how much should be
allocated to Partner AA?
a. 44,250
b. 47,500
c. 41,000
d. 41,167

PROB. 1-36 (Adapted)


Partners AA and BB have profit and loss agreement with the following provisions: salaries of
P30,000 and P45,000 for AA and BB, respectively; a bonus to AA of 10% of net income after
salaries and bonus; and interest of 10% on average capital balances of P20,000 and P35,000
for AA and BB, respectively. One-third of any remaining profits will be allocated to AA and
the balance to BB. If the partnership had net income of P22,000, how much should be
allocated to Partner AA, assuming that the provisions of the profit and loss agreement are
ranked by order of priority starting with salaries?
13,200
12,500
12,000
8,800

PROB. 1-37 (Adapted)

Luz, Vi, and Minda are partners when the partnership earned a profit of P30,000. Their
agreement provides the following regarding the allocation of profits and losses:
a. 8% interest on partners' ending capital in excess of P75,000.
b. Salaries of P20,000 for Luz and P30,000 for Vi.
c. Any balance is to be distributed 2:1:1 for Luz, Vi, and Minda, respectively.

Assume ending capital balances of P60,000, P80,000, and P100,000 for partners Luz, Vi, and
Minda, respectively. What is the amount of profit allocated for Minda, if each provision of
the profit and loss agreement is satisfied to whatever extent possible using the priority
order shown above?
a. (3,600)
b. 3,600

c. (2,000)
d. 2,000

PROB. 1-38 (Adapted)

XYZ Partnership provided for the following in their distribution of profits and losses:

First: X to receive 10% of net income up to P100,000 and 20% of the amount in excess
thereof.

Then: Y and Z are each to receive 5% of the remaining income in excess of P150,000
after X's share.

Finally: The balance is to be distributed equally to the three partners.

If the partnership earned a net income of P250,000, what is the total share of Partner X?
a. 100,000
b. 108,000
c. 110,000
d. 130,000

PROB. 1-39 (AICPA)

Hanz, Ivy, Jasper, and Kelly own a publishing company that they operate as a partnership.
Their agreement includes the following:
 Hanz will receive a salary of P20,000 and a bonus of 3% of income after all the
bonuses.
 Ivy will receive a salary of P10,000 and a bonus of 2% of income after all the
bonuses.
 All partners are to receive the following: Hanz - P5,000; Ivy - P4,500; Jasper -
P2,000; and Kelly – P4,700, representing 10% interest on their average capital
balances.
 Any remaining profits are to be divided equally among the partners.
a. How would a net loss of P40,000 would be allocated among the partners?
Hanz Ivy Jasper Kelly
a. 3,261.75 (7,169.25) (18,181.25) (17,911.25)
b. 3,450.00 (7,050.00) (19,550.00) (16,850.00)
c. 4,116.75 (6,764.25) (20,026.25) (17,326.25)
d. 45,000.00 4,500 (8,000.00) (5,300.00)

b. Assuming a profit of P40,000, how would this amount be distributed to them given the
following order of priority: Interest on invested capital, then bonuses, then salary, and
then according to profit and loss percentage?
Hanz Ivy Jasper Kelly
a. 3,261.75 (7,169.25) (18,181.25) (17,911.25)
b. 3,450.00 (7,050.00) (19,550.00) (16,850.00)
c. 4,116.75 (6,764.25) (20,026.25) (17,326.25)
d. 45,000.00 4,500 (8,000.00) (5,300.00)
PROB. 1-40 (Adapted)

On October 31, 2016, Zita and Jones formed a partnership by investing cash of P300,000
and P200,000, respectively. The partners agreed to receive an annual salary allowance of
P360,000, and to give Zita a bonus of 20% of the net income after partners' salaries, the
bonus being treated as an expense. If the profits after salaries and bonus are to be divided
equally, and the profits on December 31, 2016 after partners' salaries but before bonus of
Zita is P360,000, how much is the share of Zita in the profit?
a. 100,000
b. 120,000
c. 210,000
d. 270,000

PROB. 1-41 (AICPA)

Maxwell is trying to decide whether to accept a salary of P40,000 or salary of P25,000 plus a
bonus of 10% of net income after salaries and bonus as a means of allocating profit among
partners. Salaries traceable to the other partners are estimated to be P100,000. What
amount of income would be necessary so that Maxwell would consider choices to be equal?
a. 165,000
b. 290.000
c. 265,000
d. 305,000

PROB. 1-42 (AICPA)

A partnership has the following accounting amounts:


Sales P 700,000
Cost of goods sold 400,000
Operating expenses 100,000
Salary allocations to partners 130,000
Interest paid to banks 20,000
Partners’ drawings 80,000

What is the partnership net income (loss)?


a. 200,000
b. 180,000
c. 50,000
d. (30,000)

PROB. 1-43 (Adapted)

Alder, Benson, and Carl are capitalist partners and Denver, an industrial partner. The
partnership reported a net loss of P100,000. How much is the share of Denver in the
reported net loss?
a. 0
b. 10,000
c. 25,000
d. 100,000

PROB. 1-44 (Adapted)

If a new partner acquires a partnership interest directly from the partners rather than from
the partnership itself,
a. No entry is required.
b. The partnership assets should be revalued.
c. The existing partners' capital accounts should be reduced and the new partner's account
increased.
d. The partnership has undergone a quasi-reorganization.

PROB. 1-45 (AICPA)

Which of the following results in dissolution of a partnership?


a. The contribution of additional assets to the partnership by an existing partner.
b. The receipt of a draw by an existing partner.
c. The winding up of the partnership and the distribution of remaining assets to the
partners.
d. The withdrawal of a partner from a partnership.

PROB. 1-46 (AICPA)

When a new partner is admitted to a partnership, an original partner's capital account may
be adjusted for
a. A proportionate share of the incoming partner's investment.
b. His or her share of previously unrecorded intangible assets traceable to the original
partners.
c. His or her share of previously unrecorded intangible assets traceable to the incoming
partner.
d. None of the above.

PROB. 1-47 (AICPA)

Which of the following best characterizes the bonus method of recording a new partner's
investment in a partnership?
a. Net assets of the previous partnership are not revalued.
b. The new partner's initial capital balance is equal to his or her investment.
c. Assuming that recorded assets are properly valued, the book value of the new
partnership is equal to the book value of the previous partnership and the investment of
the new partner.
d. The bonus always results in an increase to the previous partners’ capital balances.

PROB. 1-48 (AICPA)

If goodwill is traceable to the previous partners, it is


a. Allocated among the previous partners according to their interest in capital.
b. Allocated among the previous partners only if there are not other assets to be revalued.
c. Allocated among the previous partners according to their original profit and loss sharing

percentages.
d. Not possible for goodwill to also be traceable to the incoming partner.

PROB. 1-49 (Adapted)

The goodwill and the bonus methods are two means of adjusting for differences between
the net book value and the fair market value of partnership when new partners are
admitted. Which of the following statements about these methods is correct?
a. The bonus method does not revalue assets to market values.
b. The bonus method revalues assets to market values.
c. Both methods result in the same balances in the partner capital accounts.
d. Both methods result in the same total value of partner capital account, but the
individual capital account vary.
PROB. 1-50 (AICPA)

Blau and Rubi are partners who share profits and losses in the ratio of 6:4 respectively. On
May 1, 2016, their respective capital accounts were as follows:
Blau 60,000
Rubi 50,000

On that date, Lind was admitted as a partner with one-third interest in capital, and profits
for an investment of P40,000. The new partnership began with total capital of P150,000.
Immediately after Lind's admission, Blau's capital should be
a. 50,000
b. 54,000
c. 56,667
d. 60,000

PROB. 1- 51 (AICPA)

Partnership A has an existing capital of P70,000. Two partners currently own the
partnership and split profits 50/50. A new partner is to be admitted and will contribute net
assets with a fair value of P90,000. For no goodwill or bonus (depending on whichever
method is used) to be recognized, what is the interest in the partnership granted the new
partner?
a. 33.33%
b. 50.00%
c. 56.25%

d. 75.00%

PROB. 1- 52 (AICPA)

Dunn and Grey are partners with capital account balances of P60,000 and P90,000,
respectively. They agree to admit Zorn as a partner with one-third interest in capital and
profits, for an investment of P100,000, after revaluing the assets of Dunn and Grey.
Goodwill to the original partners should be
a. 0
b. 33,333
c. 50,000
d. 66,667
PROB. 1- 53 (RPCPA)

Mitz, Marc, and Mart are partners sharing profits in the ratio of 5:3:2, respectively. As of
December 31, 2016, their capital balances were P95,000 for Mitz. P80,000 for Marc, and
P60,000 for Mart. On January 1, 2017, the partners admitted Vince as a new partner and
according to their agreement, Vince will contribute P80,000 in cash to the partnership and
also pay P10,000 for 15% of Marc's share. Vince will be given a 20% share in profits, while
the original partners' share will be proportionately the same as before. After the admission
of Vince, the total capital will be P330,000 and Vince's capital will be P70,000.
a. The total amount of goodwill to the old partners, upon he admission of Vince would be:
a. 7,000
b 15,000
.
c. 22,000
d 37,000
.

b. The balance of Marc's capital, after the admission of Vince would be:
a. 72,600
b. 74,600
c. 79,100
d. 81,100

PROB. 1 -54 (AICPA)

Ranken purchases 50% of Lark's capital interest in the K and L partnership for P22,000. If the
capital balances of Kim and Lark are P40,000 and P30,000, respectively, Ranken's capital
balance following the purchase is
a. 22,000
b. 35,000
20,000
c. 15,000

PROB. 1 - 55 (Adapted)
The following information pertains to ABC Partnership of Amor, Bing, and Cora:
Amor, capital (20%) P 200,000
Bing, capital (30%) 200,000
Cora, capital (50%) 300,000

On this date, the partners agreed to admit Dolly into the partnership. Assumine Dolly
purchased fifty percent of the partners capital and pays P500,000 to the old partners, how
would this amount be distributed to them?
a. 100,000 150,000 250,000
b. 130,000 145,000 225,000
c. 166,667 166,667 166,666
d. 150,000 150,000 200,000

PROB. 1-56 (AICPA)

The following balance sheet is presented for the partnership of A, B, and C, who share
profits and losses in the respectively ratio of 5:3:2.
Assets Liabilities and Capital
Cash 120,000 Liabilities 280,000
Other assets 1,080,000 A, capital 560,000
B, capital 320,000
C, capital 40,000
Total 1,200,000 Total 1,200,000

Assume that the assets and liabilities are fairly valued on the balance sheet, and the
partnership decided to admit D as a new partner with a one-fifth interest and no goodwill or
bonus is to be recorded. How much should D contribute in cash or other assets?
a. 147,200
b. 184,000
c. 230,000
d. 240,000

PROB. 1-57 (RPCPA)

A, B, and C are partners, who share profits and losses in the ratio of 5:3:2, respectively. They
agree to sell D 25% of their respective capital and profits and losses ratio for a total
payment directly to the partners in the amount of P140,000.
They agree that goodwill of P60,000 is to be recorded prior to the admission of D. The
condensed balance sheet of the ABC Partnership is as follows:
Cash P 60,000 Liabilities 100,000
Noncash assets 540,000 A, capital 250,000
B, capital 150,000
C, capital 100,000
Total P 600,000 Total P 600,000

The capitals of A, B, and C, respectively after payment and admission of D are:


a. 187,500 112,500 75,000
b. 210,000 126,000 84,000
c. 280,000 168,000 112,000
d. 250,000 150,000 100,000

PROB. 1 - 58 (Adapted)

Fernando and Jose are partners with capital balances of P30,000 and P70,000, respectively.
Fernando has a 30% interest in profits and losses. All assets of the partnership are at fair
market value except equipment with book value of P300,000 and fair market value of
P320,000. At this time, the partnership has decided to admit Rosa and Linda as new
partners. Rosa contributes cash of P55,000 for a 20% interest in capital and a 30% interest
in profits and losses. Linda contributes cash of P10,000 and an equipment with a fair market
value of P50,000 for a 25% interest in capital and a 35% interest in profits and losses. Linda
is also bringing special expertise and clients contact into the new partnership.

a. Using the bonus method, what is the amount of bonus?


a. 24,750
b 18,250
.
c. 14,000
d 7,500
.

b. Using the goodwill method, what is the amount of goodwill traceable to the original
partners?
a. 60,000
b. 40,000
c. 31,250
d. 28,750

PROB. 1-59 (Adapted)

The capital balances in DEA Partnership are: D, capital P60,000; E P50,000; and A, capital
P40,000 and income ratios are: 5:3:2, respectively DEAR Partnership is formed by admitting
R to the firm with cash investment of P60,000 for a 25% interest in capital. What is the
amount of bonus to be credited to A capital in admitting R?
a. 10,000
b. 7,500
c. 3,750
d. 1,500

PROB. 1- 60 (AICPA)

On June 30, 2016, the condensed balance sheet for the partnership of Eddy, Fox, and
Grimm together with their respective profit and loss sharing percentage, was as follows:
Assets, net of liabilities P320,000

Eddy, capital (50%) P160,000


Fox, capital (30%) 96,000
Grimm, capital (20%) 64,000
P320,000

a. Eddy decided to retire from the partnership and by mutual agreement is to be paid
P180,000 out of partnership funds for his interest. Total goodwill implicit in the
agreement is to be recorded. After Eddy's retirement, what are the capital balances of

the other partners?


Fox Grimm
a. 84,000 56,000
b 102,000 68,000
.
c. 108,000 72,000
d 120,000 80,000
.

Assume instead that Eddy remains in the partnership and that Hamm admitted as a new
partner with a 25% interest in the capital of the new partnership for a cash payment of
P140,000. Total goodwill implicit in the transaction is to be recorded. Immediately after
admission of Hamm, Eddy's capital account balance should be
a. 280,000
b. 210,000
c. 160,000
d. 140,000

PROB. 1-61 (Adapted)

In May 2016, Imelda, a partner of an accounting firm, decided to withdraw when the
partners' capital balances were: Mikee, P600,000; Raul, P600,000; and Imelda, P400,000. It
was agreed that Imelda is to take the partnership's fully depreciated computer with a
second hand value of P24,000 that cost the partnership P36,000. If profits and losses are
shared equally, what would be the capital balances of the remaining partners after the
retirement of Imelda?
Mikee Raul
a. 600,000 600,000
b 592,000 592,000
.
c. 608,000 608,000
d 612,000 612,000
.

PROB. 1-62 (AICPA)

The following condensed balance sheet is presented for the partnership of Alfa and Beda,
who share profits and losses in the ratio of 60:40, respectively:
Cash 45,000
Other assets 625,000
Beda, loan 30,000
700,000

Accounts payable 120,000


Alfa, capital 348,000
Beda, capital 232,000
700,000

a. The assets and liabilities are fairly valued on the balance sheet. Alfa and Beda decide to
admit Capp as a new partner with a 20% interest. No goodwill or bonus is to be
recorded. What amount should Capp contribute in cash or other assets?
a. 110,000
b. 116,000
c. 140,000
d. 145,000
b. Instead of admitting a new partner, Alfa and Beda decide to liquidate the partnership. If
the other assets are sold for P500,000, what amount of the available cash should be
distributed to Alfa?
a. 255,000
b. 273,000
c. 327,000
d. 348,000

PROB. 1-63 (Adapted)

Penny, Naty, and Mary are partners and share profits and losses equally. Each has a capital
balancer of P1,800,000. Naty retires from the partnership and receives P1,500,000. Taking
the partnership assets to be fairly stated, the entry to record Naty's retirement is
a. Naty, capital 1,800,000 (dr)
Goodwill 300,000 (cr)
Cash 1,500,000 (cr)
b. Naty, capital 1,800,000 (dr)
Partnership assets 300,000 (cr)
Cash 1,500,000 (cr)
c. Naty, capital 1,500,000 (dr)
Cash 1,500,000 (cr)
d. Naty, capital 1,800,000 (dr)
Mary, capital 150,000 (cr)
Penny, capital 150,000 (cr)
Cash 1,500,000 (cr)

PROB. 1-64 (AICPA)

On June 30, 2016, the balance sheet for the partner hip of Coll, Maduro, and Prieto,
together with their respective profit and loss ratios, were as follows:
Assets, at cost 180,000
Coll, loan 9,000
Coll, capital (20%) 42,000
Maduro, capital (20%) 39,000
Prieto, capital (60%) 90,000
Total 180,000

Coll decided to retire from the partnership. By mutual agreement, the assets are to be
adjusted to their fair value of P216,000 at June 30, 2016. It was agreed that the partnership
would pay Coll P61,200 cash for Coll's partnership interest, including Coll's loan which is to
be repaid in full. No goodwill is to be recorded. After Coll's retirement, what is the balance
of Maduro's capital account?
a. 36,450
b. 39,000
c. 45,450
d. 46,200

PROB. 1- 65 (Adapted)

On October 31, 2016, Morris retired from the partnership of Morris, Philip, and Marl. Morris
received P55,000 representing final settlement of his interest in the amount of P50,000.
Under the bonus method,
a. P5,000 was recorded as goodwill.
b. P5,000 was recorded as expense.
c. Charged P5,000 against the capital balances of Philip and Marl.
d. P55,000 was recorded as bonus.

PROB. 1 -66 (Adapted)

Peter, Queen, and Roy are partners with capital balances of P300,000, P300,000, and
P200,000, respectively; and sharing profits and losses equally. Roy is to retire and it is
agreed that he is to take certain office equipment with second hand value of P50,000 and a
note for

his interest. The office equipment carried in the books at P65,000 but brand new would cost
P80,000. Roy's acquisition of the office equipment would result in
a. Reduction in capital of P5,000 each for Peter, Queen, and Roy.
b. Reduction in capital of P7,5000 each for Peter, Queen, and Roy.
c. Reduction in capital of P15,000 for Roy.
d. Reduction in capital of P55,000 for Roy.

PROB. 1-67 (RPCPA)

N, X, and Y are partners sharing profits and losses in the ratio of 4:3:3, respectively. The
condensed balance sheet of NXY Partnership as of December 31, 2016 is:
Cash P 50,000 Liabilities P 40,000
Other assets 130,000 N, capital 60,000
X, capital 40,000
Y, capital 40,000
Total P 180,000 Total P 180,000

a. All the partners agree to admit Z as a 1/5 partner in the partnership without any
goodwill or bonus. Z shall contribute assets amounting to
a. 28,000
b. 10,000
c. 35,000
d. 60,000
b. The NXY Partnership is dissolved and liquidated by installments. The first realization of
P40,000 cash is on the sale of other assets with book value of P80,000. After payment of
the liabilities, the cash available is distributed to N, X, and Y, respectively as follows:
a. 36,000 27,000 27,000
b. 44,000 28,000 28,000
c. 16,000 12,000 12,000
d. 24,000 13,000 13,000

PROB. 1-68 (AICPA)

Gerber, Williams, and George are partners with present capital balances of P50,000,
P60,000, and P20,000, respectively. The partners share profit and losses according to the
following percentages: 60% for Gerber, 20% for Williams, and 20% for George. Larsen is to
joint the partnership upon contributing P60,000 to the partnership in exchange for a 25%
interest in

capital and a 20% interest in profits and losses. The existing assets of the original
partnership are undervalued by P22,000. The original partners will share the balance of
profits and losses in proportion to their original percentages. What would be the capital
balances of the old partners in the new partnership using the goodwill method?
Gerber Williams George
a. 63,200 64,400 24,400
b. 93,200 74,400 34,400
c. 76,800 65,600 25,600
d. 80,000 70,000 30,000

PROB. 1-69 (AICPA)

The following is the priority sequence in which liquidation proceeds will be distributed for a
partnership:
a. Partnership drawings, partnership liabilities, partnership loans, partnership capital
balances
b. Partnership liabilities, partnership loans, partnership capital balances.
c. Partnership liabilities, partnership loans, partnership drawings, partnership capital
balances.
d. Partnership liabilities, partnership capital balances, partnership loans.

PROB. 1-70 (Adapted)

In accounting for the lump-sum liquidation of a partnership, cash payments to partners


after all non-partner creditors' claims have been satisfied, but before the final cash
distribution, should be according to
a. The partners' relative profit and loss sharing ratio.
b. The final balances in partner capital accounts.
c. The partners' relative share of the gain or loss on liquidation.
d. Safe payment computations.

PROB. 1-71 (Adapted)

In a partnership liquidation, the final cash payment to the partners should be made in
accordance with the
a. Partner's profit and loss sharing ratio.
b. Balances of partners' capital accounts.
c. Ratio of the capital contributions by partners.
d. Safe payment computations.

PROB. 1-72 (AICPA)


The doctrine of marshaling of assets
a. Is applicable only if the partnership is insolvent.
b. Allows partners to first contribute personal assets to unsatisfied partnership creditors.
c. Is applicable if either the partnership is insolvent or individual partners are insolvent.
d. Amount owed to personal creditors and to the partnership for debit capital balances are
shared proportionately from the personal assets of the partners.

PROB. 1-73 (AICPA)

Cohen, Butler, and Davis are partners in a partnership and share profits and losses 50%,
30%, and 20%, respectively. The partners have agreed to liquidate the partnership and
anticipate that liquidation expenses will total P14,000. Prior to the liquidation, the
partnership balance sheet reflects the following book values:
Cash 21,000
Non-cash assets 248,000
Notes payable to Davis 32,000
Other liabilities 154,000
Cohen, capital 60,000
Butler, capital (deficit) (10,000)
Davis, capital 33,000

Assuming that the actual liquidation expenses are P14,000 and that non-cash assets are sold
for P218,000, how would the assets be distributed to partners if Butler has net personal
assets of P8,500?
Cohen Butler Davis
a. 15,500 - -
b. 21,429 - 49,571
c. 30,650 - 53,260
d. 27,500 - 52,000

PROB. 1-74 (AICPA)

The following condensed balance sheet is presented for the partnership of Axel, Barr, and
Cain, who share profits and losses in the ratio of 4:3:3, respectively:
Cash P100,000
Other assets 300,000
Total P400,000
Liabilities P150,000
Axel, capital 40,000
Barr, capital 180,000
Cain, capital 30,000
Total P400,000

The partners agreed to dissolve the partnership after selling the other asset for P200,000.
Upon dissolution of the partnership, Axel should have received
a. 0
b. 40,000
c. 60,000
d. 70,000

PROB. 1-75 (Adapted)

Because of very unprofitable operations, partners Nal, Lou, and Gee decided to dissolve the
partnership when their capital balances and profit and loss ratio were:
Nal, capital (30%) P175,000
Lou, capital (20%) 125,000
Gee, capital (50%) 175,000
Total P475,000

Upon liquidation, all of the partnership's assets are sold and sufficient cash is realized to pay
all liabilities except one for P25,000. Gee is personally insolvent but the others are capable
of meeting any indebtedness of the firm. By what amount would the capital of Nal change?
a. 7,500 decrease
b 150,000 decrease
.
c. 195,000 decrease
d No change
. PROB. 1-76 (RPCPA)

Peter and John, who share profits and losses equally, decided to liquidate their partnership
when their net assets amounted to P260,000, and capital balances of P170,000 and
P90,000, respectively. If the noncash assets were sold for amount equal to its book value,
what amount of cash should Peter and John received?
Peter John
a. 130,000 130,000
b. 170,000 90,000
c. 180,000 80,000
d. 195,000 65,000

PROB. 1-77 (Adapted)

Sammy and Michael are partners of SM Partnership sharing profits and losses equally. They
decided to terminate the partnership when their capital balances are: Sammy, P750,000;
Michael, P500,000. At this time, the partnership owes Michael P200,000, as evidenced by a
promissory note. Upon liquidation, cash of P300,000 becomes available for distribution to
the partners. In the final cash distribution, what would be the respective share of Sammy
and Michael?
Sammy Michael
a. 150,000 150,000
b. 175,000 125,000
c. 200,000 100,000
d. 275,000 25,000

PROB. 1-78 (AICPA)


The following condensed balance sheet is presented for the partnership of Smith and Jones,
who share profits and losses in the ratio of 60:40, respectively:
Other assets P 450,000
Smith, loan 20,000
P 470,000

Accounts payable P 120,000


Smith, capital 195,000
Jones, capital 155,000
P 470,000

The partners decided to liquidate the partnership. If the other assets are sold for P385,000,
what amount of the available cash should be distributed to Smith?
a. 136,000
b. 156,000
c. 159,000
d. 195,000
PROB. 1-79 (RPCPA)
The condensed balance sheet of Alex, Jay, and John as of March 31, 2016 follows:
Cash P 28,000 Liabilities P 48,000
Other assets 265,000 Alex, capital 95,000
Jay, capital 80,000
John, capital 70,000
Total P 293,000 Total P 293,000

The income and loss ratio is 50:25:25, respectively. The partners voted to dissolve their
partnership and liquidate by selling other assets in installments. P70,000 was realized on
the first cash sale of other assets with a book value of P150,000. After settlement with
creditors, all cash available was distributed to the partners. How much cash was received by
John?
a. 10,500
b. 20,000
c. 21,250
d. 32,500

PROB. 1 - 80 (Adapted)

On December 31, 2016, the partners of MNP Partnership decided to liquid their business.
Immediately before liquidation, the following condensed sheet was prepared:
Cash P 50,000 Liabilities P 375,000
Noncash assets 900,000 Nieva, capital 80,000
Perez, capital 25,000
Munoz, capital (50%) 312,500

Nieva, capital (30%) 107,500

Perez, capital (20%) 50,000

Total P 950,000 Total P 950,000

The noncash assets were sold for P400,000. Assuming Perez is the only solvent partners,
what amount of additional cash will be invested by Perez? (rounded to the nearest peso)
a. 37,143
b. 25,000
c. 5,000
d. 0
PROB. 1-81 (Adapted)

After incurring losses resulting from very unprofitable operations, the Goh Kong Wei
Partnership decided to liquidate when the partners' capital balances were:
Goh, capital (40%) P80,000
Kong, capital (40%) 130,000
Wei, capital (20%) 96,000

The non-cash assets were sold in installment. Available cash were distributed to partners in
every sale of non-cash assets. After the second sale of non-cash assets, the partners
received the same amount of cash in the distribution. And from the third sale of non-cash
assets, cash available for distribution amounts to P28,000, and unsold non-cash assets has a
book value of P12,500. Using cash priority program, what amount did Wei received in the
third installment of cash?
a. 11,600
b. 8,000
c. 5,600
d. 0

PROB. 1-82 (AICPA)

Partners Able, Baker, and Chapman, who share profit and loss equally, have the following
personal assets, personal liabilities, and partnership capital balances:
Able Baker Chapman
Personal assets P 30,000 P 80,000 P 60,000
Personal liabilities 25,000 50,000 72,000
Capital balances 50,000 (32,000) 70,000

After applying the doctrine of marshalling of assets, the capital balances of Able, Baker, and
Chapman, respectively, would be
a. 50,000 (2,000) 58,000
b. 48,000 0 58,000
c. 49,000 0 57,000
d. 34,000 0 54,000

PROB. 1-83 (Adapted)

Partner Morgan is personally insolvent, owing P600,000. Personal assets will only bring
P200,000 when liquidated. At the same time, Morgan has a credit capital balance in the
partnership of P120,000. The capital amounts of the other partners total a credit balance of
P250,000. Under the doctrine of marshalling of assets, how much the personal creditors of
Morgan can collect?
a. 120,000
b. 200,000
c. 320,000
d. 570,000

PROB. 1-84 (RPCPA)

As of December 31, the books of AME Partnership showed capital balances of: A - P40,000; M -
P25,000; and E-P5,000. The partners' profit and loss ratio was 3:2:1, respectively. The partners
decided to dissolve and liquidate. They sold all the non-cash assets for P37.000 cash. After
settlement of all liabilities amounting to P12,000, they still have P28,000 cash left for distribution.
a. The loss on the realization of the non-cash assets was
a. 40,000
b. 42,000
c. 44,000
d. 45,000

b. Assuming that any partner's capital debit balance is uncollectible, the share of A in the
P28,000 cash for distribution would be
a. 19,000
b. 18,000
c. 17,800
d. 40,000

PROB. 1-85 (RPCPA)

The balance sheet of the partnership of Salve, Galo, and Norma, who share in the profits
and losses in the ratio of 5:3:2, respectively is as follows:
Assets Liabilities and Capital
Cash 30,000 Liabilities 50,000
Other assets 320,000 Salve, capital 80,000
Galo, capital 115,000
Norma, capital 105,000
Total 350,000 Total 350,000

The partnership is liquidated by installment. The first sale of non-cash assets with a book
value of P150,000 realizes P100,000. How should the remaining cash be distributed?
Salve Galo Norma
a. 50,000 30,000 20,000
b 40,000 24,000 16,000
.
c. 0 31,000 49,000
d 0 48,000 32,000
.

PROB. 1-86 (RPCPA)

The following balance sheet is presented for the partnership of A, B, and C, who share
profits and losses in the respectively ratio of 5:3:2.
Assets Liabilities and Capital
Cash 120,000 Liabilities 280,000
Other assets 1,080,000 A, capital 560,000
B, capital 320,000
C, capital 40,000
Total 1,200,000 Total 1,200,000

Assume that the three partners decided to liquidate the partnership. If the other assets are
sold for P800,000, how should the available cash be distributed to each partner?
A B C
a. 280,000 320,000 40,000
b 324,000 236,000 16,000
.
c. 410,000 230,000 0
d 412,000 228,000 0
.

PROB. 1 - 87 (Adapted)

In the liquidation of a partnership it is necessary to (1.) distribute cash to the partners; (2.)
sell non-cash assets; (3.) allocate any gain or loss on realization to the partners; and (4.) pay
liabilities. These steps should be performed in the following order:
a. (2), (3), (4), (1)
b. (2), (3), (1), (4)
c. (3), (2), (1), (4)
d. (3), (2), (4), (1)
PROB. 1-88 (AICPA)

Partners Almond, Barney, and Colors have capital balances of P20,000, P50,000, and P90,000,
respectively. They split profits in the ratio of 2:4:4, respectively. Under a safe cash distribution
plan, one of the partners will get the following total amount in liquidation before any other
partners get anything:
a. 0
b. 15,000
c. 40,000
d. 180,000

PROB. 1 - 89 (AICPA)

The ABC Partnership has assets with book value of P240,000 and a market value of
P195,000, outside liabilities of P70,000, loans payable to Partner Able of P20,000, and
capital balances for Partners Able, Baker, and Chapman of P70,000, P30,000, and P50,000,
respectively. The partners share profits and losses equally.
a. How would the first P100,000 of available assets be distributed?
a. P70,000 to outside liabilities, P20,000 to Able, and the balance equally among
partners.
b. P70,000 to outside liabilities, and P30,000 to Able.
c. P70,000 to outside liabilities, P25,000 to Able, and P5,000 to Chapman.
d. P40,000 to Able, P20,000 to Chapman, and the balance equally among partners.
b. If all outside creditors and loans to partners had been paid. How would the balance of
the assets be distributed assuming Chapman had already received assets with a value of
P30,000?
a. Each of the partners would received P25,000.

b. Each of the partners would received P40,000.


c. Able: P70,000, Baker: P30,000, Chapman: P20,000
d. Able: P55,000, Baker: P15,000, Chapman: P5,000.

PROB. 1-90 (RPCPA)

Roy and Gil are partners sharing profits and losses in the ratio of 1:2, respectively. On July 1,
2016, they decided to form the R&G Corporation by transferring the assets and liabilities of
the partnership to the corporation in exchange for the latter's stock. The following is the
post-closing trial balance of the partnership.
Debit Credit
Cash P 45,000
Accounts receivable (net) 60,000
Inventory 90,000
Fixed assets (net) 174,000
Liablities P 60,000
Roy, capital 94,800
Gil, capital 214,200
P369,000 P369,000

It was agreed that adjustments be made to the following assets to be transferred to the
corporation:
Accounts receivable P40,000
Inventory 68,000
Fixed assets 180,600

The R&G Corporation was authorized to issue P100 par preferred stock and P10 par
common stock. Roy and Gil agreed to receive for their equity in the partnership 720 shares
of the common stock each, plus even multiples of 10% shares of preferred stock for their
remaining interests.

a. The total number of shares of preferred and common stocks issued by the corporation
in exchange for the assets and liabilities of the partnership are:
Preferred Common
a. 2,540 shares 1,500 shares
b 2,592 shares 1,440 shares
.
c. 2,642 shares 1,440 shares
d 2,642 shares 1,550 shares
.

b. The distribution of the stocks to Roy and Gil would be:


Roy Gil
Preferred Common Preferred Common
a. 785 shares 720 shares 1,384 shares 720 shares
b 773 shares 750 shares 1,843 shares 750 shares
.
c. 758 shares 720 shares 1,834 shares 720 shares
d 738 shares 720 shares 1,758 shares 720 shares
.

PROB. 1 - 91 (AICPA)

The condensed balance sheet of Adams & Gray, a partnership, at December 31, 2016,
follows:
Current assets P 250,000
Equipment (net) 30,000
Total assets P 280,000

Liabilities P 20,000
Adams, capital 160,000
Gray, capital 100,000
Total liabilities and capital P 280,000

On December 31, 2016, the fair values of the assets and liabilities were appraised at
P240,000 and P20,000, respectively, by an independent appraiser. On January 2, 2017, the
partnership was incorporated and 1,000 shares of P5 par value common stock were issued.
Immediately after the incorporation, what amount should the new corporation report as
additional paid in capital?
a. 275,000
b. 260,000
c. 215,000
d. 0
SOLUTIONS AND EXPLANATIONS

PROB. 1-1 Suggested answer (a) Limited liability

In a partnership, each partner is personally and individually liable for all partnership
liabilities. In other words, the liability of the partners in a partnership is unlimited.

PROB. 1-2 Suggested answer (c) A partnership is characterized by limited liability

Partnerships have been affected by the proprietary theory, which looks at the entity through
the eyes of the owners. Characteristics of a partnership that emphasizes that the entity is
viewed as the individual owners include the following:
a. Salaries to partners are viewed as distributions of income rather than a component of
income;
b. Unlimited liability of general partners extends beyond the entity to the individual
partners;
c. Income of the partnership is not taxed at the partnership level but rather than, is
included as part of the partners 'individual taxable income;
d. An original partnership is dissolved upon admission or withdrawal of a partner.

PROB. 1-3 Suggested answer (c) A general partner may be a secured creditor of the limited
partnership

A general partner has a voice in management and has unlimited personal liability. Anyone,
including a secured creditor of the limited partnership, may be a general partner if he/she
takes on these responsibilities.

PROB. 1-4 Suggested answer (c) A partnership is created by mere agreements of the partners

A partnership is easily formed and is relatively free from governmental regulations and
restrictions. Decisions can be made quickly on substantive matters affecting the firm,
whereas in a corporation, formal meetings with the board of directors are often needed.

PROB. 1-5 Suggested answer (a) Fair value at the date of contribution

Where a new legal entity exists, noncash assets are permitted to be recorded at its fair
market value; thus, the capital account should be credited for the current fair value of the
assets at the date of the contribution.
PROB. 1-6 Suggested answer (d) Market value, Market value

Non-cash assets contributed into the partnership should be recorded at its current fair
value.

PROB. 1-7 Suggested answer (a)

Partnership capital accounts are similar to corporate paid in capital and retained earnings,
while partnership drawing accounts are similar to corporate dividends accounts.

PROB. 1-8 Suggested answer: (c) Çes

Al capital 50,000
Ben capital (80,000 – 35,000) 45,000
Ces capital 55,000

At the date of the formation of the partnership, all assets contributed by the partners are
recorded in the books of the partnership at their fair values, and all liabilities assumed by
the partnership are recorded at their present values.

PROB. 1-9 Suggested answer (a) 75,000, 80,000, 85,000

Al Sharif Booba
Cash contribution 50,000 80,000 25,000
Store equipment 25,000
Computer 60,000
Capital balances 75,000 80,000 85,000

Noncash assets contributed by the partners into the partnership should be recorded at its
fair market values. In this case, the fair market value is the cash selling price of the
computer and the second hand value of the store equipment.
PROB. 1-10 Suggested answer: (a) 592,000 750,000

Atta Boy
Capital balances before adjustments 620,000 800,000
a. Uncollectible accounts receivable ( 20,000) ( 40,000)
b. Worthless inventories ( 6,000) ( 7,000)
c. Intangible assets written off ( 2,000) ( 3,000)
Adjusted capital balances 592,000 750,000

When assets other than cash are invested into the partnership, it is necessary for the
partners to agree upon the value of such assets. The assets are recorded in accordance with
the agreement, and the partners' capital accounts are credited for the amounts of the
respective investments. The effects of the adjustments to the capital accounts should be in
accordance with the accounting equation (Asset = Liabilities + Capital).

PROB. 1-11 Suggested answer: (b)190,000

Mary capital before adjustments 264,000


Allowance for doubtful accounts (3% ×120,000) ( 3,600)
Merchandise inventory 25,000
Prepaid expense 3,600
Accrued liabilities ( 4,000)
Mary capital after adjustments 285,000

Total partnership capital (285,000/ 3/5) 475,000


Multiply by Jane interest 2/5
Cash to be invested by Jane 190,000

Again, when assets other than cash are invested into the partnership, it is necessary for the
partners to agree upon the value of such assets. The assets are recorded in accordance with
the agreement, and the partners' capital accounts are credited for the amounts of the
respective investments. The effects of the adjustments to the capital accounts should be in
accordance with the accounting equation (Asset = Liabilities + Capital)
In this case where Jane will have an interest of 2/5, Mary should have an interest of 3/5.
Since no goodwill or bonus was mentioned in the problem, the adjusted capital of Mary
represents her 3/5 interest, which will be used as basis to determine the total partnership
capital.
PROB. 1-12 Suggested answer (b) 35,000 75,000

Roberts Smith
Cash P 20,000 P 30,000
Inventory 15,000
Building (net of P10,000 mortgage) 30,000
Furniture & equipment 15,000
P 35,000 P 75,000

Generally, individual capital accounts should be credited for the fair market value, at the
date of contribution, of the assets contributed by that partner. The partner's capital credit is
based upon the net assets contributed by the particular partner, thus the liabilities assumed
by the partnership reduced the fair market value of the building invested.

PROB. 1-13 Suggested answer: (a)18,000

The importance of proper valuation of assets invested by partners cannot be


overemphasized. In order to achieve equity, assets invested by partners should be reported
at their fair market value. Fair value is determined by making reference to the following:
cash transactions of the same or similar assets, quoted market prices, and independent
appraisals.

PROB. 1-14

a. Suggested answer (d) 60,000


Contributed Agreed Increase
Capital Capital (Decrease)
Grey 60,000 60,000 -
Redd 20,000 60,000 40,000
Total 80,000 120,000 40,000

The partnership agreement provides for equal initial capital. Thus under the goodwill
method, the capital credit for Redd should be the same as the contribution of Grey, thereby
increasing the total agreed capital to P120,000, which is P40,000 more than the total
contributed capital (goodwill).

b. Suggested answer (b) 20,000 bonus to Redd


Contributed Agreed Increase
Capital Capital (Decrease)
Grey 60,000 40,000 (20,000)
Redd 20,000 40,000 20,000
Total 80,000 80,000

The partnership agreement provides for equal initial capital. Thus under the bonus method,
the capital credit for Redd should be the same as the contribution of Grey, resulting to
P20,000 bonus from Grey to Redd.

PROB. 1-15 Suggested answer: (d) 0

Under the bonus method, assets are not revalued, instead, adjustments are made to
partnership capital accounts; consequently, unidentifiable assets are not recognized.

PROB. 1-16 Suggested answer: (b)500,000 450,000 550,000

Alex Benjie Cesar


Contributions @fair value P500,000 P800,000 P550,000
Less liabilities assumed - 350,000 -
Capital balance, 4/30/06 P500,000 P450,000 P550,000
Again, any noncash asset contributed into the partnership should be valued at the fair value
of the noncash asset contributed. Any liabilities assumed by the partnership, reduces the
partners' capital balance. As a general guideline, what is to be recorded as a credit to
partners' capital is the fair value of the net assets contributed.

PROB. 1-17 Suggested answer (d)187,500 200,000 212,500

Abel Cain Josuah


Cash contributed P 100,000 P 160,000 P 50,000
Noncash contributed 50,000 120,000
Capital balances beginning 150,000 160,000 170,000
Distribution of net income:
(150/480 × 120,000) 37,500
(160/480 × 120,000) 40,000
(170/480 × 120,000) 42,500
Capital balances, 12/31/16 P187,500 P200,000 P212,500

Noncash assets contributed by partners to form a partnership should be recorded at its fair
value. Profits and losses are divided in accordance with the agreement of the partners,
normally, the profit and loss ratio. In the absence of any agreement, profits and losses are
divided in accordance with the partners' contributed capital.

PROB 1-18 Suggested answer (b) Flat Flat

Profit (Flat) Loss (Flat)


Bonus 20% Bonus 0
Balance (2/5 × 80%) 32% P & L (2/5 × 100%) 40%
Total advantage 52% Total advantage 40%

In case of profit Flat has 52%, an advantage, and in case of loss, its only 40%, also an
advantage.

PROB. 1-19 Suggested answer (c) In all earnings or loss situation

When agreement provides for salaries without qualification, salary distribution must be
made even though profit is inadequate to cover salaries or there is a loss. In this case,
Partner A will benefit by P6,000 in all situations, whether there is a profit or loss.

PROB. 1-20 Suggested answer (c) 9,000

Vick's share of undistributed losses (30% x 30,000) 9,000


If the partners agree to distribute profits based on profit sharing ratio but are silent on loss
sharing, partnership losses will be divided based on the agreed profit sharing proportions.

PROB. 1-21 Suggested answer: (b) 15,375

Months
Date Balances Unchanged Total
January 1 P140,000 6 P 840,000
July 1 180,000 1 180,000
August 1 165,000 5 825,000
Total 12 P1,845,000

Weighted average capital (1,845,000/ 12 months) P153,750


Interest rate 10%
Interest to be credited to Simm P 15,375

When partners wish to distribute profits in terms of relative investments, the use of average
capitals, which provides for the recognition of capital changes during the period. normally
offers the most equitable method. From the data given above, partner's investments were
expressed in terms of peso-month. Under this method (peso-day, peso-month), withdrawals
and investments made during the first half of the month should be treated as if they were
made on the first day of the month, while withdrawals and investments made during the
later half of the month should be treated as if they were made on the first day of the
following month.

PROB. 1-22 Suggested answer (b) 51,667

Months
Date Balances Unchanged Total
March 1 P 50,000 3 P 150,000
June 1 70,000 3 210,000
September 1 65,000 4 260,000
Total 12 P 620,000

Annual weighted average capital (620,000/ 12) P 15,667

The partnership agreement should provide how invested capital is to be determined. Since
each partner's equity is a combination of capital and drawing account balances, partner's
drawings may be offset against their respective capital accounts for purposes of allocating
income based on invested capital. However, the agreement may also provide that only
withdrawals more than a certain limit are to be viewed as offset against capital balances.
Thus, only P5,000 excess of P10,000 limit is viewed as deduction from the capital balance.

PROB. 1-23 Suggested answer (d) In accordance with capital contribution


The ratio in which partnership profits and losses are divided is known as profit and loss ratio.
Profits and losses are divided in accordance with the agreement of the partners. In the
absence of any agreement, profits and losses are divided in accordance with the partners'
contributed capital.

PROB. 1-24 Suggested answer (d) Interest on notes to partners

The division of partnership income should be based on an analysis of the correlation


between capital and labor committed to the firm by individual partners and the income that
subsequently is generated. As a result, profits might be divided in one or more of the
following ways: 1.) according to ratio; 2.) according to the capital investments of the
partners; and 3.) according to the labor (or service) rendered by the partners. Interest on
notes to partners is a legitimate expense of a partnership.

PROB. 1-25 Suggested answer (a) Interest paid to partners based on the amount of invested
capital

Again, the division of partnership income should be based on an analysis of the correlation
between capital and labor committed to the firm by individual partners and the income that
subsequently is generated and therefore includes interest paid to partners based on the
amount of their invested capital.

PROB. 1-26 Suggested answer (c) Being characteristic of the proprietary theory

Partnerships have been affected by the proprietary theory, which looks at the entity through
the eyes of the owners. Characteristics of a partnership that emphasizes that the entity is
viewed as the individual owners include the following:
a. Salaries to partners are viewed as distributions of income rather than a component of
income;
b. Unlimited liability of general partners extends beyond the entity to the individual
partners;
c. Income of the partnership is not taxed at the partnership level but rather than, is
included as part of the partners' individual taxable income;
d. An original partnership is dissolved upon admission or withdrawal of a partner.
PROB. 1-27 Suggested answer (b) 12,000

Partnership net income or net loss is shared based on capital contribution of the partners,
unless the partnership contract specifically indicates otherwise. As a result, it is customary
to refer to the basis as the income ratio, income and loss ratio, or the profit or loss ratio.
Thus, the share of Partner C, should be P12,000 (60,000 x 20%).

PROB. 1-28

a. Suggested answer: (d) 136,543 103,457

A, capital:
Months
Date Balances Unchanged Total
January 1 P120,000 4 P 480,000
May 1 100,000 3 300,000
Aug. 1 110,000 2 220,000
Oct. 1 100,000 3 300,000
Total 12 P1,300,000

Average capital – A = P1,300,000/12 = P108,333

B, capital:
Months
Date Balances Unchanged Total
January 1 P80,000 4 P320,000
May 1 70,000 2 140,000
July 1 90,000 3 270,000
Oct 1 85,000 3 255,000
Total 12 P985,000

Average capital – B = P985,000/12 = P82,083

A P240,000 × (82,083/190,417) P136,543


B 240,000 × (82,083/190,417) 103,457
Total P240,000

Again, under this method. (peso-day, peso-month), withdrawals and investments made
during the first half of the month should be treated as if they were made on the first day of
the month, while withdrawals and investments made during the latter half of the month
should be treated as if they were made on the first day of the following month.

b. Suggested answer: (a) 121,500 118,500


A B Total
Interest on ending capital
(100,000 × 20%) P 20,000
(85,000 × 20%) P 17,000 P 37,000
Balance (equally) 101,500 101,500 203,000
Total P121,500 P118,500 P240,000

The capital contributions to be considered as the basis for the distribution of profits and
losses should be clearly defined in the agreement, because it may be based on the original
capital contribution, on the capitals at the beginning of each period, on the capitals at the
end of each period, or on the average capitals during the period

PROB. 1-29 Suggested answer: (a) 1,000 decrease

Young Zinc Total


10% interest on ave. capital
( 10% × 160,000) P16,000
(10% × 100,000) P10,000 P26,000
Balance (equally) (11,000) (11,000) (22,000)
Total P 5,000 (P1,000) P 4,000

The partnership profit before interest was P4,000, however, it resulted to a loss of P22,000
after interest. Thus, the capital balance of Zinc decreases by P1,000.

PROB. 1-30 Suggested answer: (6) 43,000 37,000

Red White Total


Salary allowances 55,000 45,000 100,000
Loss after allowance (60:40) (12,000) (8,000) (20,000)
Earnings credited to partners 43,000 37,000 80,000
The earnings before any allowance of P80,000 is reduced by the salary allowances in the
total amount of P100,000 which resulted to a loss after allowances of P20,000, because
credits to partners capital accounts are based on earnings after allowances (e.g. interest,
salary, and bonus). It should be pointed out that per partnership agreement profits should
be shared equally and losses in a 60/40 ratio, thus the loss of P20,000 was shared at 60/40
ratio.

PROB. 1-31 Suggested answer: (a) 7,000 increase

Fox Greg Howe Total


10% interest on 12,000 6,000 4,000 22,000
average capital
Salaries 30,000 20,000 50,000
Bal (equally) (35,000) (35,000) (35,000) (105,000)
Total inc.(dec.) 7,000 (29,000) (11,000) (33,000)

Again, when the partnership agreement provides without qualification that interest is to be
allowed on investment, interest must be allowed even though operations have resulted in
earnings that are less than the allowable interest or in a loss.

And when agreement also provides for salaries without qualification, salary distribution
must be made even though profit is inadequate to cover salaries or there is a loss. Interest
and salary allowances allocated to partners increase their capital balances as well as the
amount of loss. Accordingly, the amount of loss will reduce the partners' capital accounts.
The resulting loss in the total amount of P105,000 after the interest and salary allowances
was allocated among partners equally based on their agreement, that profit and loss is
divided equally.

PROB. 1-32 (c) 4,000

Net income before bonus 44,000


Less net income after bonus (44,000/110%) 40,000
Bonus (10%) 4,000
Note that the provision for bonus is 10% of income AFTER the bonus, thus the bonus is the
difference between the net income before and after the bonus.

PROB. 1-33 Suggested answer: (a) 28,600


Donn Eddy Farr Total
10% bonus to Donn 10,000 10,000
6% interest on
average capital 4,800 3,000 1,800 9,600
Balance (equally) 26,800 26,800 26,800 80,400
Total 41,600 29,800 28,600 100,000

In some instances, a managing partner is allowed a bonus that is to be based on the


earnings

of the business. The bonus is commonly stated as a percentage of profits, but the agreement
should indicate whether the percentage is to be applied to the profit determined before
deduction of the bonus or after deduction of the bonus. Based on the data provided in this
problem, the “10% bonus of the profit” was assumed to be applied to the profit before
deduction of the bonus.

PROB. 1-34 Suggested answer (d) 210,000

Adam Eve Total


Salaries 60,000 60,000 120,000
Bonus to Adam:
NY before bonus P360,000
NY after bonus
(360,000/120%) 300,000 60,000 60,000
Balance (equally) 300,000 150,000 150,000 300,000
Total 270,000 210,000 480,000

It should be pointed out that it was clearly mentioned in the problem that the P360,000 net
income is after salaries but before bonus, therefore, the net income before salaries and
bonus should be P480,000 (120,000 + 360,000).

PROB. 1-35 Suggested answer (c) 41,000

AA BB Total
Salaries 30,000 45,000 75,000
Bonus (after bonus)
NY before bonus 27,500
NY after bonus
(27,500/110%) 25,000 2,500 2,500
10% interest 2,000 3,500 5,500
Balance (1/3:3/3) 6,500 13,000 19,500
Total 41,000 61,500 102,500

One of the alternatives in profit allocations if the net income is not sufficient is to completely
satisfy all provisions of the profit and loss agreement and use the profit and loss ratios to
absorb any deficiency or additional loss caused by such action.

PROB. 1-36 Suggested answer (d) 8,800

AA BB Total
Salaries (30:45) 8,800 13,200 22,000

Another alternative in profit allocation if the net income is not sufficient is to satisfy each of
the provision to whatever extent it is possible. In other words, the allocation of salaries
would be satisfied to whatever extent possible before the allocation of interest is begun. If
the provision of the profit and loss agreement are ranked by order of priority starting with
salaries, and the total salaries amounted to P75,000, therefore the net income of P22,000,
which is insufficient, will be distributed between AA and BB based on the degree of salary
claims.

PROB. 1-37 Suggested answer: (d) 2,000

Luz Vi Minda Total


Interest
8% × 80,000-75,000 400
8% × 100,000- 2,000 2,400
75,000
Salary (20:30) 11,040 16,560 27,600
Total 11,040 16,960 2,000 30,000
Again, where income is not sufficient or an operating loss exists, two alternatives may be
employed: 1.) all provisions of the profit and loss agreement may be satisfied and any
deficiency will be absorbed using the profit and loss ratio; and 2.) each of the provision may
be satisfied to whatever extent possible. The second alternative, as applied above, requires
that provisions of the profit and loss agreement be ranked by order of priority.

PROB. 1-38 Suggested answer (b) 108,000


X Y Z Total
10% of P100,000 to X 10,000 10,000
20% of excess to X
(20% × 150,000) 30,000 30,000
5% of remaining in excess
to Y and Z
(5% × 210,000-150,000) 3,000 3,000 6,000
Balance, equally 68,000 68,000 68,000 204,000
Total 108,000 71,000 71,000 250,000

Note that the distribution of profit is based on the agreement of the partners.
PROB. 1-39

a. Suggested answer (b) 3,450 (7,050) (19,550) (16,850)


Hanz Ivy Jasper Kelly Total
Salaries 20,000 10,000 30,000
Interest 5,000 4,500 2,000 4,700 16,200
Balance
(equally) (21,550) (21,550) (21,550) (21,550) (86,200)
Total 3,450 (7,050) (19,550) (16,850) (40,000)

Based on the information provided in the problem, the profit and loss ratios are used to
absorb any deficiency or any additional loss.

b. Suggested answer (c) 20,740 12,560 2,000 4,700


Hanz Ivy Jasper Kelly Total
Interest 5,000 4,500 2,000 4,700 16,200
Bonus (3:2) 1,143 762 1,905
Salaries (20:10) 14,597 7,298 21,895
Total 20,740 12,560 2,000 4,700 40,000

Net income before bonuses 40,000


Net income after bonuses (400,000/105%) 38,095
Bonuses 1,905

If each of the provision of the profit and loss agreement are satisfied to whatever extent it is
possible based on the given order of priority, at such provision (salaries) the remaining
amount (21,895) shall be allocated using the degree of the claims.
PROB. 1-40 Suggested answer (d) 270,000
Zita Jones Total
Salaries (360,000 × 2/12 ) 5,000 4,500 16,200
Bonus 1,143 762 1,905
Balance (equally) 14,597 7,298 21,895
Total 20,740 12,560 40,000

Net income before bonus 360,000


Net income after bonus (360,000/120%) 300,000
Bonus 60,000
Note that based on the given information, the net income after salaries is P360,000
(300,000 + 60,000), which is equivalent to P480,000 net income before salaries, bonus and
distribution of balance using profit and loss ratio (equally).

PROB. 1-41 Suggested answer: (b) 290,000

Amount of bonus needed to equalize (40,000-25,000) 15,000

Net income after bonus and salaries (15,000/10%) 150,000


Multiply by 110%
Net income before bonus but after salaries 165,000
Add salaries (10,000 + 25,000) 125,000
Net income before bonus and salaries 290,000

Since the question being asked is the amount of income necessary to equalize, the
appropriate approach is to determine the amount of the difference between the two
alternatives, which is P15,000 bonus. At this point, the net income could be determined by
working back, as shown above.

PROB. 1-42 Suggested answer: (b) 180,000

Sales 700,000
Less cost of goods sold 400,000
Gross profit 300,000
Less operating expenses 100,000
Operating profit 200,000
Less interest paid to banks 20,000
Net income 180,000
Salaries, like interest on capital investments, are viewed as a means of allocating income
rather than as an expense. The drawing account is a temporary account and is periodically
closed to the partner's capital account, and has nothing to do with the computation of net
income.

PROB. 1-43. Suggested answer: (a) 0

In case there is an industrial partner, and there is no profit and loss sharing agreement, an
industrial partner shall not be liable for the losses. As to profit, the share of an industrial
partner shall be that which is just and equitable under the circumstances. In order for an
industrial partner be liable for the losses, there should be an expressed stipulation to that
effect.

PROB. 1-44 Suggested answer (c) The existing partner's capital should be reduced and the new
partner's account increased

When a new partner deals directly with an existing partner or partners rather than with the
partnership entity, the acquisition price is paid to the selling partner/s and not to the
partnership itself. The partnership records the redistribution of capital interests by
transferring all or a portion of the seller's capital to the new partner's capital account but
does not record the transfer of any asset or consideration.

PROB. 1-45 Suggested answer (d) Withdrawal of a partner from a partnership

Dissolution is the change in the relation of the partners caused by any partner ceasing to be
associated in the carrying on as distinguished from the winding up of the business.
Generally, a partnership is dissolved upon the death, withdrawal, admission, or bankruptcy
of an individual partner (owner).

PROB. 1-46 Suggested answer (b) His or her share of previously unrecorded intangible assets
traceable to the original partner

Accounting changes in the ownership of a partnership is influenced heavily by the legal


concept of dissolution. When there is a change in the ownership structure, the original
partnership is dissolved and most often a new partnership is created. This dissolution and
subsequent creation of a partnership indicate that a new legal entity has been created and
accounting should measure properly the initial contributions of capital being made to the
new partnership.

PROB. 1-47 Suggested answer (c)Assuming that recorded assets are properly valued, the book
value of the new partnership is equal to the book value of the previous partnership and the
investment of the new partner.

Under the bonus method, total contributed capital of the old and new partner is equal to
the total agreed capital (total capital of the new partnership).
PROB. 1-48 Suggested answer (c) Allocated among the previous partners according to their
original profit and loss sharing percentages

Unrecorded goodwill also may be identifiable. If there are no differences between the fair
market value and book value of recorded assets, the new partner's willingness to pay more
than the proportionate book value of the new entity indicates that goodwill existed prior to
the new partner's admission. If this intangible asset could have been sold prior to the
admission of the partner, the realized profit would have been allocated to the old partners.
Therefore, the goodwill is recorded and allocated to the old partners according to their
profit and loss ratio.

PROB. 1-49 Suggested answer (a)The bonus method does not revalue assets to market values.

When an incoming partner's contribution is different from that indicated by the book values
of the original partnership, the admission of the partner typically is recorded by either the
bonus method or the goodwill method. Both methods permit the assumption that there is
unrecorded goodwill to be recognized. However, the use of either method does not prevent
the recognition of differences between the book value and fair market value of recorded net
assets.
The bonus method strictly follows the principle that net assets should be recorded at
historical cost and simply readjusts capital accounts and makes no changes in existing
assets accounts. While, the goodwill method emphasizes the legal significance of a change
in the ownership structure of a partnership and revalues assets to adjust the total value of
partnership capital.

PROB. 1-50 Suggested answer: (b) 54,000


Contribute Agreed Increase
Capital Capital (Decrease)
Old partners P 110,000 P 100,000 (P10,000)
New partners 40,000 (1/3) 50,000 10,000
Total P 150,000 P 150,000 -

Blau’s capital before admission of Lind P 60,000


Less share in bonus to Lind (10,000 × 60%) 6,000
Blau’s capital after admission of Lind P 54,000

When a partnership is in urgent need of additional funds or the partners move desire the
services of a certain individual, a new partner may be admitted with the provision that (a)
part of the capitals of the old partners shall be allowed as a bonus to the new partner, or (b)
goodwill shall be established and credited to the new partner.
When the total contributed capital is equal to the agreed capital, there is bonus. In this case,
the amount by which the interest allowed to the new partner exceeds his investment may be
considered as bonus contributed by the old partners. The bonus is deducted from the
capitals of the old partners based in their original profit and loss ratio.

PROB. 1 - 51 Suggested answer (c) 56.25%

Capital contributed by the new partner 90,000


Divided by total contributions (70,000 + 90,0000) 160,000
New partner’s interest 56.25%
Since no goodwill or bonus to be recognized, the capital contribution is equal to the capital
credited to partners.

PROB. 1-52 Suggested answer: (c) 50,000

Contributed Agreed Increase


Capital Capital (Decrease)
Old partners P 150,000 P 200,000 P 50,000
New partners 100,000 (1/3) 100,000 -
Total P 250,000 P 300,000 P 50,000
When a partnership has operated with considerable success, the partners may admit a new
partner with the provision that (a) part of the new partner's investment shall be allowed as
a bonus to the old partners, or (b) partnership goodwill shall be established and credited to
the old partners. When the total agreed capital is more than the contributed capital, there is
goodwill. Since the combined capitals of the old partners was increase to P200,000, the
increase in capital of P50,000 should be recognized as goodwill and distributed to them
using their original profit and loss ratio.

PROB. 1-53
a. Suggested answer: (b) 15,000

Contributed Agreed Increase


Capital Capital (Decrease)
Old partners
[235,000-(15% × 223,000 260,000 37,000
80,000)
New partners
[80,000+(15% × 80,000) 92,000 70,000 (22,000)
Total 315,000 330,000 15,000 Again,
when
the total agreed capital is more than the contributed capital, there is goodwill and this
amount of P15,000 will be distributed to the old partners using their original profit and loss
ratio. However, in the above computations, it should be pointed out that aside from
goodwill, the new partner provides bonus to old partners in the amount of P22,000
(decrease in the new partner's capital).

b. Suggested answer: (c) 79,100

Marc’s capital before Vince’s admission 80,000


Interest purchased by Vince (15% × 80,000) (12,000)
Share in goodwill (30% × 15,000) 4,500
Share in bonus (30% × 22,000) 6,600
Marc’s capital after Vince’s admission 79,100

When an incoming partner purchases a portion or all of the interest of one or more of the
original partners, the partnership assets remains unchanged and the related amount paid
for the purchase should not be recorded in the books of the partnership for this is regarded
as a personal transaction between the selling partner/s and the buyer.
Bonus to old partners is recorded by making transfer from the contributed capital of the
new partner to the old partners' capital accounts, which gives the new partners a capital
credit less than his actual investment. While, Goodwill to old partners should be recognized
by a debit to Goodwill account, and the resulting credit should be to old partners' capital
accounts allocated based on their profit and loss ratio.
PROB. 1-54 Suggested answer (d) 15,000

Ranken’s capital (50% × 30,000) 15,000

When a new partner deals directly with an existing partner or partners rather than with the
partnership entity, the acquisition price is paid to the selling partner/s and not to the
partnership itself. The partnership records the redistribution of capital interests by
transferring all or a portion of the seller's capital to the new partner's capital account but
does not record the transfer of any asset or consideration.

PROB. 1-55 Suggested answer (b) 130,000 145,000 225,000.

Amor Bing Cora


Interest purchased (1/2) 100,000 100,000 150,000
Excess payment (15,000) P&L 30,000 45,000 75,000
Total 130,000 145,000 225,000

A new partner may be admitted to the partnership by acquiring all or part of the capital
interest of one or more existing partners in exchange for some consideration. The
partnership records the redistribution of capital interest by transferring all or a portion of
the seller's capital to the new partner's capital account but does not record the transfer of
any asset. Any difference between the amounts paid by the new partner, which is not
recorded in the books of the partnership, is allocated to the selling partners based on their
profit or loss ratio.

PROB. 1-56 Suggested answer (c) 230,000

Total capital of the old partnership


(560,000 + 320,000 + 40,000) 920,000
Divide by profit and loss (old partnership) 4/5
Total capital of the new partnership 1,150,000
Multiply by profit and loss of D 1/5
Required contribution by D 230,000

If the book value of the original partnership's net assets approximates fair market value or
no bonus, no goodwill to be recognized, the incoming partner's contribution would be
expected to be equal to the portion of the equity that the new partner is acquiring.

PROB. 1-57 Suggested answer: (b) 210,000 126,000 84,000

A B C
Capital balances P250,000 P150,000 P100,000
Share in goodwill 5:3:2 30,000 18,000 12,000
Adjusted capital balances 280,000 168,000 112,000
¼ interest purchased (70,000) (42,000) (28,000)
Capital balances after D P210,000 P126,000 P 84,000

Again, when an incoming partner purchases a portion or all of the interest of one or more of
the original partners, this may be regarded as a transfer from the capital account/s of the
seller/s to that of the buyer, and any amount paid for the purchase should not be recorded
in the books of the partnership, therefore, the partnership assets remained the same. The
one-fourth interest purchased by D was based on the adjusted capitals of the old partners
(after goodwill) because of the stipulation in the problem that goodwill is to be recorded
prior to the admission of D.

PROB. 1-58

a. Suggested answer: (b) 18,250

Contributed Agreed Increase


Capital Capital (Decrease)
Old partners P 100,000 P 200,000 P 18,250
New partners 115,000 (45%) 100,000 (18,250)
Total P 215,000 P 215,000 -

The goodwill and bonus methods are two means of adjusting for differences between the
net book value and the fair market value of partnership when new partners are admitted.
The goodwill method revalues assets to market value; while the bonus method does not
revalue assets to market value. Further, under the bonus method, the total contributed
capital is equal to the total agreed capital.

b. Suggested answer: (c) 31,250


Contributed Agreed Increase
Capital Capital (Decrease)
Old partners P 100,000 P 151,250 P 51,250
New partners 115,000 (45%) 123,750 8,750
Total P 215,000 P 275,000 P 60,000

Total increase in capital 60,000


Less undervalued equipment (320,000 – 300,000) 20,000
Balance 40,000
Goodwill to Linda 8,750
Goodwill to original partners 31,250

Again, when there is a difference between the book value and fair market value of the
partnership when new partners are admitted, the goodwill method revalues assets to
market value. Ordinarily, to determine the new capital of the partnership, contributed
capital of the new partner may be divided by his interest in capital. In this case, where Linda
will be provided with goodwill for bringing her expertise and clients contact to the
partnership, the capital of Rosa was used instead, because it serves as concrete basis with
no goodwill involved, in determining the new capital of the partnership. Thus, the new
capital of the partnership is P275,000 (55,000/20%).

PROB. 1-59 Suggested answer: (d) 1,500

Contributed Agreed Increase


Capital Capital (Decrease)
Old partners 150,000 157,500 7,500
New partners 60,000 52,500 25% (7,500)
Total 210,000 210,000 -

Bonus to A (7,500 × 20%) 1,500

Under the bonus method of admitting a new partner into the partnership, the total
contributed capital (including that of the new partner) is equal to the new partnership
capital. Accordingly, any bonus to the old partners shall be allocated using their old profit
and loss ratio.

PROB. 1-60
a. Suggested answer: (c) 108,000 72,000
Goodwill to be paid to Eddy (P180,000 – 160,000) P20,000
Divide by Eddy P&L 50%
Total goodwill P40,000
Fox Grimm
Capital balance before goodwill P 96,000 P64,000
Goodwill: (40,000 × 30%) 12,000
(40,000 × 30%) 8,000
Capital balance after goodwill P108,000 P72,000

Since the problem identified that total goodwill implicit in the agreement is to be recorded,
the excess of the amount received by Eddy over his capital balance represent his share in the
total goodwill to be recognized. Accordingly, Fox and Grimm will share in the total goodwill
based on their respective profit and loss percentage.

b. Suggested answer: (b) 210,000


Contributed Agreed Increase
Capital Capital (Decrease)
Old partners P 320,000 P 420,000 P 100,000
New partners 140,000 (25%) 140,000 -
Total P 460,000 P 560,000 P 100,000

Eddy’s capital before goodwill P 160,000


Share in goodwill (50% × 100,000) 50,000
Eddy’s capital after goodwill P 210,000

Again, goodwill is the excess of total agreed capital over the contributed capital. In this
case, the amount of P100,000 represents goodwill to old partners, which will be divided
based on their respective profit and loss ratio.

PROB. 1-61 Suggested answer: (c) 608,000 608,000

Mikee Raul
Capital balance before withdrawal 600,000 600,000
Distribution of gain on realization (24,000/ 3) 8,000 8,000
Capital balance after withdrawal 608,000 608,000

When a partner withdraws, he may receive an amount equal, more than or less than his
interest. The interest of the withdrawing partner is measured by his capital balances
adjusted by the distribution of profit or loss from operations, and changes in valuation of all
assets and liabilities. Thus, their capital balances will be increased by their respective share
in the realization of noncash asset with a fair value different from its book value at the date
of withdrawal.

PROB. 1-62

a. Suggested answer: (d) 145,000


Total capital [(348,000 + 232,000)/ 80%] 725,000
Capp’s interest × 20%
Cash or other assets to be contributed by Capp 230,000

It should be pointed out that the problem clearly state that no bonus or goodwill is to be
recognized, thus the total capital of the old partners was used as the basis in computing the
total capital of the partnership.

b. Suggested answer: (b) 273,000


Alfa Beda
Capital balances 348,000 232,000
Beda, loan (30,000)
Total interest 348,000 202,000
Loss on realization (75,000) (50,000)
Cash to be distributed 273,000 152,000

Beda, loan account was appropriately presented as an asset (receivable of the partnership
from Beda), therefore it will reduce the capital balance of Beda. And loss on realization
should be charged to the partners' capital. In partnership liquidation, cash is distributed
based on the capital balances of the partners after any adjustments. It should be pointed
out that the total amount of cash to be distributed between partners equals the total
partners’ capital after adjustments.

PROB. 1-63 Suggested answer (d)

Note that the capital balance of the retiring partner is P1,800,000 and was paid P1,500,000,
this situation will result to the recognition of bonus to the remaining partners from the
retiring

partner. Thus the difference of P300,000 was allocated to the remaining partner based on
their profit and loss agreement (equally) by crediting their respective capital account.

PROB. 1-64 Suggested answer: (c) 45,450

Coll Maduro Prieto


Capital balances before
Retirement of Coll P42,000 P39,000 P 90,000
Coll, loan 9,000
Adjustments of assets 2:2:6
(216,000 – 180,000) 7,200 7,200 21,600
Total interest 58,200 46,200 111,600
Less payment to Coll 61,200
Balance (3,000) 46,200 111,600
Bonus to Coll 2:6 3,000 (750) (2,250)
Capital balances after
Retirement of Coll P 0 P45,450 P109,350

Again, when partner withdraws from a partnership, adjustment of assets to its fair market
value should be made. Total interest of the withdrawing partner must be determined and be
compared with the amount paid. Since the problem stated that the withdrawing partner is
selling his interest to the partnership and no goodwill is to be recorded, the resulting
difference between the total interest and the amount paid represents the bonus provided by
the remaining partners to the withdrawing partner.

PROB. 1-65 Suggested answer (c)

Under the bonus method, the excess of the amount paid by the partnership to the retiring
partner shall be absorbed by the remaining partners based on their existing profit and loss
ratio.

PROB. 1–66 Suggested answer (d) Reduction in capital of P55,000 for Roy.

Peter Queen Roy


Second hand value taken 50,000
Loss on realization
(65,000 – 50,000) (equally) 5,000 5,000 5,000
Total reduction in capital 5,000 5,000 55,000

PROB. 1-67
a. Suggested answer: (c) 35,000
Total partnership capital 175,000
Multiply by 1/5
Assets to be contributed to Z 35,000

Again, if there is no bonus or goodwill to be recognized, total partnership capital may be


computed using the capital accounts of the old partners as the base, as shown above.

b. Suggested answer: (d) 24,000 13,000 13,000


N X Y
Capital balances P 60,000 P 40,000 P 40,000
Loss on realization
(80,000 – 40,000) 4:3:3 (16,000) (12,000) (12,000)
Possible loss
(130,000 – 80,000) 4:3:3 (20,000) (15,000) (15,000)
Cash distribution P 24,000 P 13,000 P 13,000

In installment liquidation, to insure an equitable distribution of cash to the partners,


considerable care is required. Usually, the statement of liquidation, is supported by the
Schedule of Safe Payments, wherein each installment on cash is distributed as if no more
cash is forthcoming. Thus, cash is distributed to a partner only if he has an excess credit
balance in his partnership interest after absorption of his share of the maximum possible
loss that may occur, which consists of the assumed unrealizable value of the remaining
noncash assets, cash withheld for payment of anticipated liquidation expenses and
unrecorded liabilities that may arise, and any additional loss that may also accrue to the
partners when a debit balance in any of the capital accounts results from the allocations of
possible loss. Should the liquidation extend over a long period of time, these calculations
may become frequent such that it may be desirable to prepare in advance an installment
distribution plan, known as Cash Priority Program.

PROB. 1-68 Suggested answer (d) 80,000 70,000 30,000

Gerber Williams George Total


Capital contributions 50,000 60,000 20,000 130,000
Asset revaluation 13,200 4,400 4,400 22,000
Total 63,200 64,400 24,400 152,000
Goodwill to old partners 16,800 5,600 5,600 28,000
Total capital 80,000 70,000 30,000 180,000
Contributed Agreed Increase
Capital Capital (Decrease)
Old partners 152,000 180,000 28,000
New partners 60,000 60,000 25%
Total 212,000 240,000 28,000

Again, under the goodwill method total contributed capital is less than the total agreed
capital.

PROB. 1-69 Suggested answer (b)

In the event of liquidation, subject to any agreement to the contrary, the following sequence
of payments should be observed:
1. Amounts owed to creditors other than partners;
2. Amounts owed to partners other than for capital and profits (i.e. partners' loans to the
partnership;
3. Amounts owed to partners as capital
4. Amounts owed to partners as profits not currently closed to partners’ capital accounts.
PROB. 1-70 Suggested answer (d) Safe payment computations

A lump-sum liquidation is one in which all of the assets are sold in bulk and all of the
creditors' claims are satisfied before a single liquidating distribution is made to the partners.
Because assets are sold in bulk, there is a tendency to realize greater losses than if the
assets were sold over a period of time. Therefore, payments to partners should be
distributed safely.

PROB. 1-71 Suggested answer (b) Balances of partners' capital accounts

The four basic steps to partnership liquidation are:


1. Net income or loss up to the date of liquidation should be allocated to the partners'
capital accounts based on their P&L ratio.
2. The gain or loss realized from the sale of noncash assets should be allocated to the
partners' capital accounts based on their P&L ratio.
3. Creditors' claims, including liquidation expenses or anticipated future claims should be
paid.
4. Remaining cash is distributed to partners in accordance with the balance in their capital
accounts, and not the P&L ratio.
PROB. 1-72 Suggested answer (c)

The provisions that call for the contribution of personal assets to a liquidating partnership
illustrate the characteristics of unlimited liability. However, such personal liability depends
on the legal doctrine of marshaling of assets. This doctrine, which is applied when the
partnership and/or one or more of the partners are insolvent, states that:
a. Partnership assets are first available for the payment of partnership debts. Any excess
assets are available for payment of the individual partner's debts, but only to the extent
of the partner's interest in the capital of the partnership;
b. Personal assets of a partner are applied against personal debts, ranked in the order of
priority as follows: 1.) Amounts owed to persona creditors; 2.) Amounts owed to
partnership creditors; and 3.) Amounts owed to partners by way of contribution.

PROB. 1-73 Suggested answer (d) 27,500 0 52,000

Cohen Butler Davis Total


Capital balances 60,000 (10,000) 33,000 83,000
Notes payable to Davis 32,000 32,000
Actual liquidation exps. (7,000) (4,200) (2,800) (14,000)
Loss on realization
(248,000 – 218,000) (15,000) (9,000) (6,000) (30,000)
Addt’l contribution – B 8,500 8,500
Balances 38,000 (14,700) 56,200 79,500
Absorption – Butler (10,500) 14,700 (4,200) -
Cash payments 27,500 - 52,000 79,500

Liquidation expenses and loss on realization are charged against the partners capital
accounts. The liability of partners in a partnership is unlimited, thus, the creditors of the
partnership may go after the partners to the extent of their personal assets. However, it
should be noted that under the doctrine of marshalling of assets, personal assets should be
applied first to personal obligations and any excess shall be applied to the unliquidated
partnership obligation.
PROB. 1-74 Suggested answer: (a) 0

Axel Barr Cain


Capital balances
Before liquidation P40,000 P180,000 P30,000
Loss on realization 4:3:3
(200,000 – 300,000) 40,000 30,000 30,000
Capital balances
After liquidation P 0 P150,000 P 0

Since all assets are distributed at one point in time rather than installments, this represents
simple liquidation. As assets are converted into cash, any differences between the book
values and the amounts realized represent gains or losses to be divided among partners in
the profit and loss ratio. Such gains and losses are carried to the capital accounts. The
capital balances then becomes the basis for settlement.

PROB. 1-75 Suggested answer: (c) 195,000 decrease

Nal Lou Gee


Capital balances
Before liquidation P175,000 P125,000 P175,000
Distribution of loss 3:2:5
(475,000 + 25,000) (150,000) (100,000) (250,000)
Balances 25,000 25,000 (75,000)
Absorption of Gee (3:2) (45,000) (30,000) 75,000
Capital balances (P20,000) (P5,000) -

Distribution of loss P 150,000


Absorption of Gee 45,000
Decrease in capital of Nal P 195,000

Note that upon liquidation, all of the partnership's assets are sold and sufficient cash is
realized to pay all claims except one for P25,000, therefore, the partnership incurred loss
from realization, which will eventually reduce the capital balances of the partners. In
addition, the deficiency of an insolvent partner is simply eliminated by absorption, thus
increasing the decrease in capital account of Nal to P195,000, as shown above. Incidentally,
since Nal and Lou are solvent, additional cash of P20,000 and P5,000, respectively will be
invested by them.

PROB. 1-76 Suggested answer: (b) 170,000 90,000

Given that noncash assets were sold for an amount equal to its book value, therefore, no
gain or loss was realized from the sale of noncash assets. Thus, the partners will receive an
amount equal to their respective capital balances before liquidation.

PROB. 1-77 Suggested answer (b) 175,000, 125,000

Total capital of partnership (750,000 + 500,000) 1,250,000


Add liability (Notes payable) 200,000
Total assets 1,450,000
Less available cash 300,000
Loss on realization 1,150,000

Sammy Michael
Capital balances before liquidation 750,000 500,000
Notes payable to Michael 200,000
Total interest 750,000 700,000
Loss on realization (equally) (1,150,000) 575,000 575,000
Capital balances after cash distribution 175,000 125,000

Loss on realization is the difference between the total assets (equal to the total capital and
liabilities) and the amount realized from its sale.

PROB. 1-78 Suggested answer: (a) 136,000

Smith Jones
Capital balances P 195,000 P 155,000
Smith, loan (Dr) (20,000)
Total interest 175,000 155,000
Loss on realization (60:40)
(450,000 – 385,000) (39,000) (26,000)
Cash available for partners P 136,000 P 129,000

Again, since all assets are distributed at one point in time rather than installments, this
represents simple liquidation or total liquidation. The loan account was presented as an
asset

of the partnership, therefore, it is a receivable on the part of the partnership, and


eventually, reduces the capital balance of the partner concerned.

PROB. 1-79 Suggested answer: (b) 20,000

Alex Jay John


Capital balances P 95,000 P 80,000 P 70,000
Loss on realization
(150,000 – 70,000) 2:1:1 (40,000) (20,000) (20,000)
Possible loss
(265,000 – 150,000) 2:1:1 (57,500) (28,750) (28,750)
Balances (2,500) 31,250 21,250
Absorption of Alex 1:1 2,500 (1,250) (1,250)
Cash distribution P 0 P 30,000 P 20,000

Again, in installment liquidation, to insure an equitable distribution of cosh to the partners,


considerable care is required. Usually, the statement of liquidation is supported by the
Schedule of Safe Payments, wherein each installment of cash is distributed as if no more
cash is forthcoming.

Thus, cash is distributed to a partner only if he has an excess credit balance in his
partnership interest after absorption of his share of the maximum possible loss that may
occur, which consists of the assumed unrealizable value of the remaining noncash assets,
cash withheld for payment of anticipated liquidation expenses and unrecorded liabilities
that may arise, and any additional loss that may also accrue to the partners when a debit
balance in any of the capital accounts results from the allocations of possible loss.

PROB. 1-80 Suggested answer: (b) 25,000

Munoz Nieva Perez


Capital balances
Before liquidation 312,500 107,500 50,000
Loan balances 80,000 25,000
Total interest 312,500 187,500 75,000
Loss on realization (5:3:2)
(900,000 – 400,000) (250,000) (150,000) (100,000)
Balances (deficiency) 62,500 37,500 (25,000)

The loan balances were presented in the liability section of the condensed balance sheet,
thus, these should be treated as obligations of the partnership and eventually increase the
interests of the partners. Since Perez is the only solvent partner, while other partners incur
no deficiency, Perez will invest additional cash equal to his own deficiency.

PROB. 1-81 Suggested answer: (c) 5,600

When the liquidation period extend over a long period of time, calculations of safe
payments to partners may become frequent and bothersome such that it may be desirable
to prepare in advance an installment distribution plan, called Cash Priority Program. This
program, which is an alternative to Schedule of Safe Payments, permits the partners to
determine how cash should be safely distributed when it becomes available. Using this
program, the first priority of payments should be provided to the partner with the highest
loss absorption balance. The amount to be paid could be determined by multiplying the
excess loss absorption balance of partner over another by his profit and loss percentage.
The process should be continually done until the loss absorption balances of the partners are
equal, in which case, distribution of available cash should be made according to their profit
and loss percentage.

Since there are three partners in the firm (Goh, Kong, and Wei), in the third priority of
payments, the amount to be received by Wei is P5,600 (P28,000 x 20%), because at this
point, their loss absorption balances are equal.

PROB. 1-82 Suggested answer: (c) 49,000, 0, 57,000

Able Baker Chapman


Capital balances 50,000 (32,000) 70,000
Personal assets applied
to capital deficiency
(80,000 – 50,000) 30,000
Capital balances applied
to personal liabilities
(72,000 – 60,000) (12,000)
Balances 50,000 (2,000) 58,000
Absorption of Baker (1,000) 2,000 (1,000)
Balances after applying
the doctrine of
marshalling of assets. 49,000 0 57,000

The doctrine of marshalling of assets is applied when the partnership and/or one or more of
the partners are insolvent. It provides that partnership assets are first available for payment
of partnership debts. Any excess is applied to individual partners' debt to the extent of his
interest. While, personal assets are applied against personal debts, any excess is applied to
partnership creditors, then to partner's debit capital balance.

PROB. 1-83 Suggested answer: (c) 320,000

Personal assets 200,000


Add capital credit balance 120,000
Total amount due to personal creditors 320,000

Note that the capital accounts of Morgan and other partners have credit balances,
therefore, the partnership has no deficient partner. Accordingly there is no possible
absorption that will reduce the capital of Morgan, thus the personal creditors of Morgan
can collect, not only his personal assets but to the extent of his entire capital balance.

PROB. 1 – 84

a. Suggested answer (b) 42,000


Total capital before liquidation (40,000 + 25,000 + 5,000) 70,000
Less: Cash left for distribution 28,000
Loss on realization of the non-cash assets 42,000

Upon payment to partners in the event of liquidation, the amount of cash available for
distribution is always equal to the total partners' capital.

b. Suggested answer (c) 17,800


A M E
Capital balances before liquidation 40,000 25,000 5,000
Loss on realization (3:2:1) (42,000) (21,000) (14,000) (7,000)
Balances 19,000 11,000 (2,000)
Absorption of E (3:2) (1,200) (800) 2,000
Cash payments to A and B 17,800 10,200 -

In the event of liquidation, all liquidation expenses and gains or losses from conversion of
partnership assets must be allocated to the partners before assets actually are distributed
to

the individual partners. Failure to consider these factors may result in the premature
distribution of assets to a partner. Furthermore, generally accepted accounting principle
states that partners should contribute assets to the partnership to the extent of their debit
balances. However, if such contribution is not possible because of special personal or legal
considerations, the debit balance will be viewed as a realization loss and allocated
according to the remaining partners' profit or loss ratio.

PROB. 1-85 Suggested answer (c) 0 31,000 49,000

Salve Galo Norma


Capital balances before liquidation 80,000 115,000 105,000
Loss on realization (50,000) (5:3:2) (25,000) (15,000) (10,000)
Balances 55,000 100,000 95,000
Less: Possible loss (170,000) (5:3:2) (85,000) (51,000) (34,000)
Balances (30,000) 49,000 61,000
Absorption of Salve (3:2) 30,000 (18,000) (12,000)
Safe payment to partners - 31,000 49,000
To avoid the problem associated with premature payments, installment payments may be
made to partners only after anticipating all liabilities, possible losses, and liquidation
expenses. The allocation of the assumed loss is allocated among the partners according to
their profit and loss ratio. The allocation of the assumed loss could produce debit balances in
partners' capital accounts, and these balances are treated as being uncollectible. Therefore,
the assumed debit capital balances are allocated to those partners with credit balances
according to their profit and loss ratio.

PROB. 1-86 Suggested answer (c) 410,000 230,000 0

A B C
Capital balances before 560,000 320,000 40,000
liquidation
Loss on realization (280,000) (140,000) (84,000) (56,000)
(5:3:2)
Balances 420,000 236,000 (16,000)
Absorption of C (5:3) (10,000) (6,000) 16,000
Payment to A and B 410,000 230,000 -
Again, generally accepted accounting principle states that partners should contribute assets
to the partnership to the extent of their debit balances. However, if such contribution is not
possible because of special personal or legal considerations, the debit balance will be viewed
as a realization loss and allocated according to the remaining partners' profit or loss ratio.

PROB. 1-87 Suggested answer (a) 2, 3, 4, 1

Unlike a dissolution where the partnership continues its business purpose liquidation results
in the partnership's ending or terminating its business. The process of liquidation consists of
the conversion of partnership assets into a distributable form and the distribution of these
assets to creditors and owners. All liquidation expenses and gains or losses from conversion
of partnership assets also must be allocated to the partners before assets actually are
distributed to the individual partners. Failure to consider these factors may result in the
premature or incorrect distribution of assets to a partner.

PROB. 1-88 Suggested answer (c) 40,000

Almond Barney Colors


Total Interest 20,000 50,000 90,000
Divide by P&L 20% 40% 40%
Loss absorption balance 100,000 125,000 225,000
Priority 1 – Colors (100,000)
Balances 100,000 125,000 125,000
Priority 2 – Colors & Barney (25,000) (25,000)
Balances (P & L) 100,000 100,000 100,000

Since the question being asked is "one of the partners will get .... before any other partners
get anything", therefore, it is the partner under priority no. 1 (Colors). Colors shall received,
under priority no. 1, P40,000 (100,000 x 40%)

PROB. 1-89

a. Suggested answer (b) P70,000 outside liabilities, P30,000 Able


Able Baker Chapman
Total Interest 90,000 30,000 50,000
Divide by P&L (equally) 1/3 1/3 1/3
Loss absorption balance 270,000 90,000 150,000
Priority 1 – Able (120,000)
Balances 150,000 90,000 150,000
Priority 2 – Able & Chapman (60,000) (60,000)
Balances (P & L) 90,000 90,000 90,000
Priority 3 – P&L

Payments by priority Able Baker Chapman


Priority 1 (120,000 × 1/3) 40,000
Priority 2 (60,000 × 1/3) 20,000 20,000

Cash Liability Able Baker Chapman


Total 100,000
Liability (70,000) 70,000
Balance 30,000
Loan – A (20,000) 20,000
Balance 10,000
Priority 1 (10,000) 10,000
Total 70,000 30,000

The Partnership Law provides, that in the event of partnership liquidation, available cash
shall be distributed in the following order: first: to outside creditors, second: to inside
creditors (partners' loan), and third: to owners (partners' capital accounts).

b. Suggested answer (d) Able: 55,000, Baker: 15,000, Chapman: 5,000


Able Baker Chapman
Total Interest (excluding loan) 70,000 30,000 50,000
Divide by P&L (equally) 1/3 1/3 1/3
Loss absorption balance 210,000 90,000 150,000
Priority 1 – Able (60,000)
Balances 150,000 90,000 150,000
Priority 2 – Able & Chapman (60,000) (60,000)
Balances 90,000 90,0000 90,000
Priority 3 – P&L

Payments by priority Able Baker Chapman


Priority 1 (60,000 × 1/3) 20,000
Priority 2 (60,000 × 1/3) 20,000 20,000

Cash Able Baker Chapman


MV of assets 195,000
Liability (70,000)
Able, loan (20,000)
Balance 105,000
Priority 1 (20,000) 20,000
Balance 85,000
Priority 2 (40,000) 20,000 20,000
Balance 45,000
Priority 3 (45,000) 15,000 15,000 15,000
Total 55,000 15,000 35,000
Less: Asset taken
by chapman 30,000
balance 55,000 15,000 5,000

PROB. 1 – 90
a. Suggested answer: (b) 2,592 1,440
Roy Gil Total
Capital before adjustment P94,800 P214,200 P309,000
Accounts receivable (6,667) (13,333) (20,000)
Inventory (7,333) (14,667) (22,000)
Fixed assets 2,200 4,400 6,600
Total stock to be issued 83,000 190,600 273,600
Common stock (720 × 10) 7,200 7,200 14,400
Preferred stock P100 par P75,800 P183,400 P259,200

Share to be issued: Roy Gil Total


Common stock 720 720 1,440
Preferred stock 758 1,834 2,592
In order to secure the advantages found in the corporate form of business organization, the
partners may decide to incorporate. Upon incorporation, the corporation will act to acquire
the net assets of the partnership in exchange for its stock. The stock received by the
partnership is distributed to the partners in settlement of their equities. The corporation
therefore takes over the assets and assumes the liabilities of the partnership; the
partnership is dissolved and the partners now become stockholders in the newly formed
corporation. The allocated common shares and preferred shares were computed by dividing
the amount of each stock by its respective par value, as shown above.

b. Suggested answer: (c) 758 720 1,834 720


See above computations in question "a".

PROB. 1-91 Suggested answer: (c) 215,000

Net assets at fair value (P240,000 – 20,000) P 220,000


Less par value of stock issued (1,000 × 5) 5,000
Additional paid in capital 215,000

The incorporation of a partnership results in the formation of a new accounting (and legal)
entity. This means that the partnership must adjust its records up to the date of
incorporation. Among others, the partnership's books are adjusted to reflect the fair market
value of the partnership assets and the present value of partnership liabilities. In addition,
when capital stock were issued, capital stock account should be credited at par or stated
value, any excess over par or stated value must be credited to additional paid in capital.

You might also like